1,2,3,4,5,6,7,8,9,12,31 - Test 1 review

¡Supera tus tareas y exámenes ahora con Quizwiz!

During which phase of the nurse-patient relationship can the nurse anticipate that identified patient issues will be explored and resolved? a. Preorientation b. Orientation c. Working d. Termination

ANS: C During the working phase, the nurse strives to assist the patient in making connections among dysfunctional behaviors, thinking, and emotions and offers support while alternative coping behaviors are tried.

Josefina Juarez, aged 36 years, comes to the mental health clinic where you work after being referred by her primary care provider. Josefina came to live in the United States from Brazil 5 years ago. She is now a single mother to 6 children, ages 2 to 15, following the death of her husband last year. During the initial intake assessment, Josefina tells you her problem is that she has headaches and backaches "almost every day" and "can't sleep at night." She shakes her head no and looks away when asked about anxiety or depression and states she does not know why she was referred to the mental health clinic. You recognize that Josefina may be exhibiting: A. regression. B. somatization. C. enculturation. D. assimilation.

B. Somatization is described as experiencing and expressing emotional or psychological distress as physical symptoms. Regression is a defense mechanism meaning to begin to function at a lower or previous level of functioning. Enculturation refers to how cultural beliefs, practices, and norms are communicated to its members. Assimilation refers to a situation in which immigrants adapt to and absorb the practices and beliefs of a new culture until these customs are more natural than the ones they learned in their homeland.

Which source of healing might be most satisfactory to a client who believes his illness is caused by spiritual forces? A. Acupuncture B. Dietary change C. Cleansings D. Herbal medicine

C. Rituals, cleansings, prayer, and even witchcraft may be the treatment expectation of a client who believes his illness is caused by spiritual forces.

People who have an indigenous worldview A. see themselves as spiritual and believe that they are linked with all other living things. B. focus on the articulation of individual needs and ideas. C. view the self as an extension of cosmic energy that is repeatedly reborn. D. are concerned with being part of a harmonious community.

D. Clients with an indigenous worldview are interested in connectedness and being in harmony with others. They have little interest in personal goals and autonomy.

Which nursing intervention below is part of the scope of an advanced practice psychiatric/mental health nurse only? a. Coordination of care b. Health teaching c. Milieu therapy d. Psychotherapy

ANS: D Psychotherapy is part of the scope of practice of an advanced practice nurse. The distracters are within a staff nurse's scope of practice.

A patient with a long history of hypertension and diabetes now develops confusion. The health care provider wants to make a differential diagnosis between Alzheimer's disease and multiple infarcts. Which diagnostic procedure should the nurse expect to prepare the patient for first a. Skull x-rays b. Computed tomography (CT) scan c. Positron-emission tomography (PET) d. Single-photon emission computed tomography (SPECT)

B A CT scan shows the presence or absence of structural changes, including cortical atrophy, ventricular enlargement, and areas of infarct, information that would be helpful to the health care provider. PET and SPECT show brain activity rather than structure and may occur later. See relationship to audience response question.

Select the example of a tort. A.The plan of care for a patient is not completed within 24 hours of the patient's admission. B.A nurse gives a PRN dose of an antipsychotic drug to an agitated patient because the unit is short-staffed. C.An advanced practice nurse recommends hospitalization for a patient who is dangerous to self and others. D.A patient's admission status changed from involuntary to voluntary after the patient's hallucinations subside.

B A tort is a civil wrong against a person that violates his or her rights. Giving unnecessary medication for the convenience of staff controls behavior in a manner similar to secluding a patient; thus, false imprisonment is a possible charge. The other options do not exemplify torts.

An obese patient has a diagnosis of schizophrenia. Medications that block which receptors would contribute to further weight gain? a. H1 b. Acetylcholine c. 5 HT2 d. Gamma-aminobutyric acid (GABA)

A H1 receptor blockade results in weight gain, which is undesirable for an obese patient. Blocking of the other receptors would have little or no effect on the patient's weight.

The nurse administers a medication that potentiates the action of gamma-aminobutyric acid (GABA). Which effect would be expected? a. Reduced anxiety b. Improved memory c. More organized thinking d. Fewer sensory perceptual alterations

A Increased levels of GABA reduce anxiety. Acetylcholine and substance P are associated with memory enhancement. Thought disorganization is associated with dopamine. GABA is not associated with sensory perceptual alterations.

38. A newly hospitalized patient experiencing psychosis says, "Red chair out town board." Which term should the nurse use to document this finding? a. Word salad b. Neologism c. Anhedonia d. Echolalia

ANS: A Word salad (schizophasia) is a jumble of words that is meaningless to the listener and perhaps to the speaker as well, because of an extreme level of disorganization.

The nurse should assess a patient taking a drug with anticholinergic properties for inhibited function of the: a. Parasympathetic nervous system b. Sympathetic nervous system c. Reticular activating system d. Medulla oblongata

A Acetylcholine is the neurotransmitter found in high concentration in the parasympathetic nervous system. When anticholinergic drugs inhibit acetylcholine action, blurred vision, dry mouth, constipation, and urinary retention commonly occur.

A psychiatric nurse best applies the ethical principle of autonomy by: A.exploring alternative solutions with a patient, who then makes a choice. B.suggesting that two patients who were fighting be restricted to the unit. C.intervening when a self-mutilating patient attempts to harm self. D.staying with a patient demonstrating a high level of anxiety.

A Autonomy is the right to self-determination, that is, to make one's own decisions. By exploring alternatives with the patient, the patient is better equipped to make an informed, autonomous decision. The distracters demonstrate beneficence, fidelity, and justice.

A patient in alcohol rehabilitation reveals to the nurse, "I feel terrible guilt for sexually abusing my 6-year-old before I was admitted." Select the nurse's most important action. A. Anonymously report the abuse by phone to the local child protection agency B. Reply, "I'm glad you feel comfortable talking to me about it." C. File a written report with the agency's ethics committee. D. Respect nurse-patient relationship confidentiality.

A Laws regarding child abuse reporting discovered by a professional during the suspected abuser's alcohol or drug treatment differ by state. Federal law supersedes state law and prohibits disclosure without a court order except in instances in which the report can be made anonymously or without identifying the abuser as a patient in an alcohol or drug treatment facility.

A nurse cares for four patients who are receiving clozapine, lithium, fluoxetine, and venlafaxine, respectively. With which patient should the nurse be most alert for problems associated with fluid and electrolyte imbalance? The patient receiving: a. lithium (Lithobid) b. clozapine (Clozaril) c. fluoxetine (Prozac) d. venlafaxine (Effexor)

A Lithium is a salt and known to alter fluid and electrolyte balance, producing polyuria, edema, and other symptoms of imbalance. Patients receiving clozapine should be monitored for agranulocytosis. Patients receiving fluoxetine should be monitored for acetylcholine block. Patients receiving venlafaxine should be monitored for heightened feelings of anxiety.

A drug causes muscarinic receptor blockade. The nurse will assess the patient for: a. dry mouth b. gynecomastia c. pseudoparkinsonism d. orthostatic hypotension

A Muscarinic receptor blockade includes atropine-like side effects, such as dry mouth, blurred vision, and constipation. Gynecomastia is associated with decreased prolactin levels. Movement defects are associated with dopamine blockade. Orthostatic hypotension is associated with 1 antagonism.

A nurse is concerned that an agency's policies are inadequate. Which understanding about the relationship between substandard institutional policies and individual nursing practice should guide nursing practice? A. Agency policies do not exempt an individual nurse of responsibility to practice according to professional standards of nursing care. B. Agency policies are the legal standard by which a professional nurse must act and therefore override other standards of care. C. Faced with substandard policies, a nurse has a responsibility to inform the supervisor and discontinue patient care immediately. D. Interpretation of policies by the judicial system is rendered on an individual basis and therefore cannot be predicted.

A Nurses are professionally bound to uphold standards of practice regardless of lesser standards established by a health care agency or a state. Conversely, if the agency standards are higher than standards of practice, the agency standards must be upheld. The courts may seek to establish the standard of care through the use of expert witnesses when the issue is clouded.

In order to release information to another health care facility or third party regarding a patient diagnosed with a mental illness, the nurse must obtain: A. a signed consent by the patient for release of information stating specific information to be released. B. a verbal consent for information release from the patient and the patient's guardian or next of kin. C. permission from members of the health care team who participate in treatment planning. D. approval from the attending psychiatrist to authorize the release of information.

A Nurses have an obligation to protect patients' privacy and confidentiality. Clinical information should not be released without the patient's signed consent for the release.

A person in the community asks, "Why aren't people with mental illness kept in state institutions anymore?" Select the nurse's best response. A. "Less restrictive settings are available now to care for individuals with mental illness." B. "There are fewer persons with mental illness, so less hospital beds are needed." C. "Most people with mental illness are still in psychiatric institutions." D. "Psychiatric institutions violated patients' rights."

A The community is a less restrictive alternative than hospitals for treatment of persons with mental illness. The distracters are incorrect and part of the stigma of mental illness.

Insurance will not pay for continued private hospitalization of a mentally ill patient. The family considers transferring the patient to a public hospital but expresses concern that the patient will not get any treatment if transferred. Select the nurse's most helpful reply. A. "By law, treatment must be provided. Hospitalization without treatment violates patients' rights." B. "All patients in public hospitals have the right to choose both a primary therapist and a primary nurse." C. "You have a justifiable concern because the right to treatment extends only to provision of food, shelter, and safety." D. "Much will depend on other patients, because the right to treatment for a psychotic patient takes precedence over the right to treatment of a patient who is stable."

A The right to medical and psychiatric treatment was conferred on all patients hospitalized in public mental hospitals with the enactment of the federal Hospitalization of Mentally Ill Act in 1964.

The laboratory report for a patient taking clozapine (Clozaril) shows a white blood cell count of 3000 mm3. Select the nurse's best action. a. Report the results to the health care provider immediately. b. Administer the next dose as prescribed. c. Give aspirin and force fluids. d. Repeat the laboratory test.

A These laboratory values indicate the possibility of agranulocytosis, a serious side effect of clozapine therapy. These results must be immediately reported to the health care provider, and the drug should be withheld. The health care provider may repeat the test, but in the meantime, the drug should be withheld. Caution: This question requires students to apply previous learning regarding normal and abnormal values of white blood cell counts.

A patient's sibling says, "My brother has a mental illness, but the doctor ordered a functional magnetic resonance image (fMRI) test. That test is too expensive and will just increase the hospital bill." Select the nurse's best responses. Select all that apply. a. "Sometimes there are physical causes for psychiatric symptoms. This test will help us understand whether that is the situation." b. "Some mental illnesses are evident on fMRIs. This test will give information to help us plan the best care for your brother." c. "Tell me more about what kinds of tests your brother's health insurance plan covers." d. "It sounds like you do not truly believe your brother had a mental illness." e. "It would be better for you to discuss your concerns with the doctor."

A, B The correct responses provide information to the sibling. Modern imaging techniques are important tools in assessing molecular changes in mental disease and marking the receptor sites of drug action, which can help in treatment planning. Psychiatric symptoms can be caused by anatomical or physiologic abnormalities. There is no evidence of denial in the sibling's comment. The nurse can answer this question rather than referring it to the physician. It would be inappropriate to discuss finances with the patient's sibling.

An individual is experiencing problems with memory. Which of these structures are most likely to be involved in this deficit? Select all that apply. a. Amygdala b. Hippocampus c. Occipital lobe d. Temporal lobe e. Basal ganglia

A, B, D The frontal, parietal, and temporal lobes of the cerebrum play a key role in the storage and processing of memories. The amygdala and hippocampus also play roles in memory. The occipital lobe is predominantly involved with vision. The basal ganglia influence integration of physical movement, as well as some thoughts and emotions.

A nurse prepares to administer a second-generation antipsychotic medication to a patient diagnosed with schizophrenia. Additional monitoring for adverse effects will be most important if the patient has which co-morbid health problems? Select all that apply. a. Parkinson's disease b. Grave's disease c. Hyperlipidemia d. Osteoarthritis e. Diabetes

A, C, E Antipsychotic medications may produce weight gain, which would complicate care of a patient with diabetes, and increase serum triglycerides, which would complicate care of a patient with hyperlipidemia. Parkinson's disease involves changes in transmission of dopamine and acetylcholine, so these drugs would also complicate care of this patient. Osteoarthritis and Grave's disease should have no synergistic effect with this medication.

A patient repeatedly stated, "I'm stupid." Which statement by that patient would show progress resulting from cognitive behavioral therapy? a. "Sometimes I do stupid things." b. "Things always go wrong for me." c. "I always fail when I try new things." d. "I'm disappointed in my lack of ability."

ANS: A "I'm stupid" is a cognitive distortion. A more rational thought is "Sometimes I do stupid things." The latter thinking promotes emotional self-control. The distracters reflect irrational or distorted thinking. This item relates to an audience response question.

Which idea held by the nurse would best promote the provision of culturally competent care? A. Western biomedicine is one of several established healing systems. B. Some individuals will profit from use of both Western and folk healing practices. C. Use of cultural translators will provide valuable information into health-seeking behaviors. D. Need for spiritual healing is a concept that crosses cultural boundaries.

A. A nurse who holds this belief would be likely be open to a variety of established interventions. In truth, nurses cannot apply a standard model of assessment, diagnosis, and intervention to all clients with equal confidence. This leads to culturally irrelevant interventions.

The nurse assesses the wellness beliefs and values of a client from another culture best when asking A. "What do you think is making you ill?" B. "When did you first feel ill?" C. "How can I help you get better?" D. "Did you do something to cause the illness?"

A. Asking the client to suggest reasons for the illness will best provide an opportunity to become familiar with general beliefs and values the client holds regarding his wellness.

According to the Western scientific view of health, illness is the result of A. pathogens. B. energy blockage. C. spirit invasion. D. soul loss.

A. Disease has a cause (e.g., pathogens, toxins) that creates the effect; disease can be observed and measured.

The psychiatric nurse planning and implementing care for culturally diverse clients should understand A. holistic theory. B. systems theory. C. adaptation theory. D. political power theory.

A. In most cultures a holistic perspective prevails, one without separation of mind and body.

In the Eastern tradition, disease is believed to be caused by A. fluctuations in opposing forces. B. outside influences. C. members' disobedience. D. adoption of Western beliefs.

A. In the Eastern tradition, disease is believed to be caused by fluctuations in opposing forces, the yin-yang energies.

Which statement best explains the term "worldview"? A. Beliefs and values held by people of a given culture about what is good, right, and normal. B. Ideas derived from the major health care system of the culture about what causes illness. C. Cultural norms about how, when, and to whom illness symptoms may be displayed. D. Valuing one's beliefs and customs over those of another group.

A. A worldview is a system of thinking about how the world works and how people should behave in the world and toward each other. It is from this view that people develop beliefs, values, and the practices that guide their lives.

29. A patient diagnosed with schizophrenia begins to talks about "macnabs" hiding in the warehouse at work. The term "macnabs" should be documented as: a. a neologism. b. concrete thinking. c. thought insertion. d. an idea of reference.

ANS: A A neologism is a newly coined word having special meaning to the patient. "Macnabs" is not a known common word. Concrete thinking refers to the inability to think abstractly. Thought insertion refers to thoughts of others are implanted in one's mind. Ideas of reference are a type of delusion in which trivial events are given personal significance.

A patient says, "Please don't share information about me with the other people." How should the nurse respond? a. "I will not share information with your family or friends without your permission, but I share information about you with other staff." b. "A therapeutic relationship is just between the nurse and the patient. It is up to you to tell others what you want them to know." c. "It depends on what you choose to tell me. I will be glad to disclose at the end of each session what I will report to others." d. "I cannot tell anyone about you. It will be as though I am talking about my own problems, and we can help each other by keeping it between us."

ANS: A A patient has the right to know with whom the nurse will share information and that confidentiality will be protected. Although the relationship is primarily between the nurse and patient, other staff needs to know pertinent data. The other incorrect responses promote incomplete disclosure on the part of the patient, require daily renegotiation of an issue that should be resolved as the nurse-patient contract is established, and suggest mutual problem solving. The relationship must be patient centered. See relationship to audience response question.

27. The family of a patient diagnosed with schizophrenia is unfamiliar with the illness and family's role in recovery. Which type of therapy should the nurse recommend? a. Psychoeducational b. Psychoanalytic c. Transactional d. Family

ANS: A A psychoeducational group explores the causes of schizophrenia, the role of medication, the importance of medication compliance, support for the ill member, and hints for living with a person with schizophrenia. Such a group can be of immeasurable practical assistance to the family. The other types of therapy do not focus on psychoeducation.

A nurse makes an initial visit to a homebound patient diagnosed with a serious mental illness. A family member offers the nurse a cup of coffee. Select the nurse's best response. a. "Thank you. I would enjoy having a cup of coffee with you." b. "Thank you, but I would prefer to proceed with the assessment." c. "No, but thank you. I never accept drinks from patients or families." d. "Our agency policy prohibits me from eating or drinking in patients' homes."

ANS: A Accepting refreshments or chatting informally with the patient and family represent therapeutic use of self and help to establish rapport. The distracters fail to help establish rapport.

17. A patient diagnosed with schizophrenia is very disturbed and violent. After several doses of haloperidol (Haldol), the patient is calm. Two hours later the nurse sees the patient's head rotated to one side in a stiff position, the lower jaw thrust forward, and drooling. Which problem is most likely? a. An acute dystonic reaction b. Tardive dyskinesia c. Waxy flexibility d. Akathisia

ANS: A Acute dystonic reactions involve painful contractions of the tongue, face, neck, and back. Opisthotonos and oculogyric crisis may be observed. Dystonic reactions are considered emergencies requiring immediate intervention. Tardive dyskinesia involves involuntary spasmodic muscular contractions that involve the tongue, fingers, toes, neck, trunk, or pelvis. It appears after prolonged treatment. Waxy flexibility is a symptom seen in catatonic schizophrenia. Internal and external restlessness, pacing, and fidgeting are characteristics of akathisia.

A patient would benefit from therapy in which peers as well as staff have a voice in determining patients' privileges and psychoeducational topics. Which approach would be best? a. Milieu therapy b. Cognitive therapy c. Short-term dynamic therapy d. Systematic desensitization

ANS: A Milieu therapy is based on the idea that all members of the environment contribute to the planning and functioning of the setting. The distracters are individual therapies that do not fit the description.

A community mental health nurse has worked with a patient for 3 years but is moving out of the city and terminates the relationship. When a novice nurse begins work with this patient, what is the starting point for the relationship? a. Begin at the orientation phase. b. Resume the working relationship. c. Initially establish a social relationship. d. Return to the emotional catharsis phase.

ANS: A After termination of a long-term relationship, the patient and new nurse usually have to begin at ground zero, the orientation phase, to build a new relationship. If termination is successfully completed, the orientation phase sometimes progresses quickly to the working phase. Other times, even after successful termination, the orientation phase may be prolonged.

A patient says, "I've done a lot of cheating and manipulating in my relationships." Select a nonjudgmental response by the nurse. a. "How do you feel about that?" b. "I am glad that you realize this." c. "That's not a good way to behave." d. "Have you outgrown that type of behavior?"

ANS: A Asking a patient to reflect on feelings about his or her actions does not imply any judgment about those actions, and it encourages the patient to explore feelings and values. The remaining options offer negative judgments.

Which technique is most applicable to aversion therapy? a. Punishment b. Desensitization c. Role modeling d. Positive reinforcement

ANS: A Aversion therapy is akin to punishment. Aversive techniques include pairing of a maladaptive behavior with a noxious stimulus, punishment, and avoidance training.

33. A nurse asks a patient diagnosed with schizophrenia, "What is meant by the old saying 'You can't judge a book by looking at the cover.'?" Which response by the patient indicates concrete thinking? a. "The table of contents tells what a book is about." b. "You can't judge a book by looking at the cover." c. "Things are not always as they first appear." d. "Why are you asking me about books?"

ANS: A Concrete thinking refers to an impaired ability to think abstractly. Concreteness is often assessed through the patient's interpretation of proverbs. Concreteness reduces one's ability to understand and address abstract concepts such as love or the passage of time. The incorrect options illustrate echolalia, an unrelated question, and abstract thinking.

18. An acutely violent patient diagnosed with schizophrenia receives several doses of haloperidol (Haldol). Two hours later the nurse notices the patient's head rotated to one side in a stiffly fixed position, the lower jaw thrust forward, and drooling. Which intervention by the nurse is indicated? a. Administer diphenhydramine (Benadryl) 50 mg IM from the PRN medication administration record. b. Reassure the patient that the symptoms will subside. Practice relaxation exercises with the patient. c. Give trihexyphenidyl (Artane) 5 mg orally at the next regularly scheduled medication administration time. d. Administer atropine sulfate 2 mg subcut from the PRN medication administration record.

ANS: A Diphenhydramine, trihexyphenidyl, benztropine, and other anticholinergic medications may be used to treat dystonias. Swallowing will be difficult or impossible; therefore, oral medication is not an option. Medication should be administered immediately, so the intramuscular route is best. In this case, the best option given is diphenhydramine.

A Category V tornado hits a community, destroying many homes and businesses. Which nursing intervention would best demonstrate compassion and caring? a. Encouraging persons to describe their memories and feelings about the event b. Arranging transportation to the local community mental health center c. Referring a local resident to a community food bank d. Coordinating psychiatric home care services

ANS: A Disaster victims benefit from telling their story. Nurses show compassion by listening and offering hope. The distracters identify other aspects of psychological first aid and services on the mental health continuum.

Which statement shows a nurse has empathy for a patient who made a suicide attempt? a. "You must have been very upset when you tried to hurt yourself." b. "It makes me sad to see you going through such a difficult experience." c. "If you tell me what is troubling you, I can help you solve your problems." d. "Suicide is a drastic solution to a problem that may not be such a serious matter."

ANS: A Empathy permits the nurse to see an event from the patient's perspective, understand the patient's feelings, and communicate this to the patient. The incorrect responses are nurse- centered (focusing on the nurse's feelings rather than the patient's), belittling, and sympathetic.

While talking with a patient diagnosed with major depression, a nurse notices the patient is unable to maintain eye contact. The patient's chin lowers to the chest, while the patient looks at the floor. Which aspect of communication has the nurse assessed? a. Nonverbal communication b. A message filter c. A cultural barrier d. Social skills

ANS: A Eye contact and body movements are considered nonverbal communication. There are insufficient data to determine the level of the patient's social skills or whether a cultural barrier exists.

A nurse and patient discuss a problem the patient has kept secret for many years. Afterward the patient says, "I feel so relieved that I finally told somebody." Which term best describes the patient's feeling? a. Catharsis b. Superego c. Cognitive distortion d. Counter-transference

ANS: A Freud initially used talk therapy, known as the cathartic method. Today we refer to catharsis as "getting things off our chests." The superego represents the moral component of personality.

A patient is having difficulty making a decision. The nurse has mixed feelings about whether to provide advice. Which principle usually applies? Giving advice: a. is rarely helpful. b. fosters independence. c. lifts the burden of personal decision making. d. helps the patient develop feelings of personal adequacy.

ANS: A Giving advice fosters dependence on the nurse and interferes with the patient's right to make personal decisions. It robs patients of the opportunity to weigh alternatives and develop problem-solving skills. Furthermore, it contributes to patient feelings of personal inadequacy. It also keeps the nurse in control and feeling powerful.

Inpatient hospitalization for persons with mental illness is generally reserved for patients who: a. present a clear danger to self or others. b. are noncompliant with medication at home. c. have limited support systems in the community. d. develop new symptoms during the course of an illness.

ANS: A Hospitalization is justified when the patient is a danger to self or others, has dangerously decompensated, or needs intensive medical treatment. The distracters do not necessarily describe patients who require inpatient treatment.

A community mental health nurse has worked for months to establish a relationship with a delusional, suspicious patient. The patient recently lost employment and could no longer afford prescribed medications. The patient says, "Only a traitor would make me go to the hospital." Select the nurse's best initial intervention. a. With the patient's consent, contact resources to provide medications without charge temporarily. b. Arrange a bed in a local homeless shelter with nightly on-site supervision. c. Hospitalize the patient until the symptoms have stabilized. d. Ask the patient, "Do you feel like I am a traitor?"

ANS: A Hospitalization may damage the nurse-patient relationship, even if it provides an opportunity for rapid stabilization. If medication is restarted, the patient may possibly be stabilized in the home setting, even if it takes a little longer. Programs are available to help patients who are unable to afford their medications. A homeless shelter is inappropriate and unnecessary. Hospitalization may be necessary later, but a less restrictive solution should be tried first, since the patient is not dangerous. A yes/no question is non-therapeutic communication.

A patient diagnosed with major depression has lost 20 pounds in one month, has chronic low self-esteem, and a plan for suicide. The patient has taken an antidepressant medication for 1 week. Which nursing intervention has the highest priority? a. Implement suicide precautions. b. Offer high-calorie snacks and fluids frequently. c. Assist the patient to identify three personal strengths. d. Observe patient for therapeutic effects of antidepressant medication.

ANS: A Implementing suicide precautions is the only option related to patient safety. The other options, related to nutrition, self-esteem, and medication therapy, are important but are not priorities.

During a one-on-one interaction with the nurse, a patient frequently looks nervously at the door. Select the best comment by the nurse regarding this nonverbal communication. a. "I notice you keep looking toward the door." b. "This is our time together. No one is going to interrupt us." c. "It looks as if you are eager to end our discussion for today." d. "If you are uncomfortable in this room, we can move someplace else."

ANS: A Making observations and encouraging the patient to describe perceptions are useful therapeutic communication techniques for this situation. The other responses are assumptions made by the nurse.

A nurse explains to the family of a mentally ill patient how a nurse-patient relationship differs from social relationships. Which is the best explanation? a. "The focus is on the patient. Problems are discussed by the nurse and patient, but solutions are implemented by the patient." b. "The focus shifts from nurse to patient as the relationship develops. Advice is given by both, and solutions are implemented." c. "The focus of the relationship is socialization. Mutual needs are met, and feelings are shared openly." d. "The focus is creation of a partnership in which each member is concerned with growth and satisfaction of the other."

ANS: A Only the correct response describes elements of a therapeutic relationship. The remaining responses describe events that occur in social or intimate relationships.

Which technique will best communicate to a patient that the nurse is interested in listening? a. Restating a feeling or thought the patient has expressed. b. Asking a direct question, such as "Did you feel angry?" c. Making a judgment about the patient's problem. d. Saying, "I understand what you're saying."

ANS: A Restating allows the patient to validate the nurse's understanding of what has been communicated. Restating is an active listening technique. Judgments should be suspended in a nurse-patient relationship. Close-ended questions such as "Did you feel angry?" ask for specific information rather than showing understanding. When the nurse simply states that he or she understands the patient's words, the patient has no way of measuring the understanding.

13. A community mental health nurse wants to establish a relationship with a very withdrawn patient diagnosed with schizophrenia. The patient lives at home with a supportive family. Select the nurse's best plan. a. Visit daily for 4 days, then every other day for 1 week; stay with patient for 20 minutes, accept silence; state when the nurse will return. b. Arrange to spend 1 hour each day with the patient; focus on asking questions about what the patient is thinking or experiencing; avoid silences. c. Visit twice daily; sit beside the patient with a hand on the patient's arm; leave if the patient does not respond within 10 minutes. d. Visit every other day; remind the patient of the nurse's identity; encourage the patient to talk while the nurse works on reports.

ANS: A Severe constraints on the community mental health nurse's time will probably not allow more time than what is mentioned in the correct option; yet, important principles can be used. A severely withdrawn patient should be met "at the patient's own level," with silence accepted. Short periods of contact are helpful to minimize both the patient's and the nurse's anxiety. Predictability in returning as stated will help build trust. An hour may be too long to sustain a home visit with a withdrawn patient, especially if the nurse persists in leveling a barrage of questions at the patient. Twice-daily visits are probably not possible, and leaving after 10 minutes would be premature. Touch may be threatening. Working on reports suggests the nurse is not interested in the patient.

A 26-month-old displays negative behavior, refuses toilet training, and often says, "No!" Which stage of psychosexual development is evident? a. Oral b. Anal c. Phallic d. Genital

ANS: B The anal stage occurs from age 1 to 3 years and has as its focus toilet training and learning to delay immediate gratification. The oral stage occurs between birth and 1 year. The phallic stage occurs between 3 and 5 years, and the genital stage occurs between age 13 and 20 years.

Select the example of tertiary prevention. a. Helping a person diagnosed with a serious mental illness learn to manage money b. Restraining an agitated patient who has become aggressive and assaultive c. Teaching school-age children about the dangers of drugs and alcohol d. Genetic counseling with a young couple expecting their first child

ANS: A Tertiary prevention involves services that address residual impairments, with a goal of improved independent functioning. Restraint is a secondary prevention. Genetic counseling and teaching school-age children about substance abuse and dependence are examples of primary prevention.

A patient is suspicious and frequently manipulates others. To which psychosexual stage do these traits relate? a. Oral b. Anal c. Phallic d. Genital

ANS: A The behaviors in the stem develop as the result of attitudes formed during the oral stage, when an infant first learns to relate to the environment. Anal-stage traits include stinginess, stubbornness, orderliness, or their opposites. Phallic-stage traits include flirtatiousness, pride, vanity, difficulty with authority figures, and difficulties with sexual identity. Genital-stage traits include the ability to form satisfying sexual and emotional relationships with members of the opposite sex, emancipation from parents, a strong sense of personal identity, or the opposites of these traits.

Which patient is the best candidate for brief psychodynamic therapy? a. An accountant with a loving family and successful career who was involved in a short extramarital affair b. An adult with a long history of major depression who was charged with driving under the influence (DUI) c. A woman with a history of borderline personality disorder who recently cut both wrists d. An adult male recently diagnosed with anorexia nervosa

ANS: A The best candidates for psychodynamic therapy are relatively healthy and well-functioning individuals, sometimes referred to as the "worried well," who have a clearly circumscribed area of difficulty and are intelligent, psychologically minded, and well-motivated for change. Patients with psychosis, severe depression, borderline personality disorders, and severe character disorders are not appropriate candidates for this type of treatment.

Which finding best indicates that the goal "Demonstrate mentally healthy behavior" was achieved? A patient: a. sees self as capable of achieving ideals and meeting demands. b. behaves without considering the consequences of personal actions. c. aggressively meets own needs without considering the rights of others. d. seeks help from others when assuming responsibility for major areas of own life.

ANS: A The correct response describes an adaptive, healthy behavior. The distracters describe maladaptive behaviors.

A 4-year-old grabs toys from siblings and says, "I want that now!" The siblings cry, and the child's parent becomes upset with the behavior. According to Freudian theory, this behavior is a product of impulses originating in which system of the personality? a. Id b. Ego c. Superego d. Preconscious

ANS: A The id operates on the pleasure principle, seeking immediate gratification of impulses. The ego acts as a mediator of behavior and weighs the consequences of the action, perhaps determining that taking the toy is not worth the mother's wrath. The superego would oppose the impulsive behavior as "not nice." The preconscious is a level of awareness. This item relates to an audience response question.

When a female Mexican American patient and a female nurse sit together, the patient often holds the nurse's hand. The patient also links arms with the nurse when they walk. The nurse is uncomfortable with this behavior. Which analysis is most accurate? a. The patient is accustomed to touch during conversation, as are members of many Hispanic subcultures. b. The patient understands that touch makes the nurse uncomfortable and controls the relationship based on that factor. c. The patient is afraid of being alone. When touching the nurse, the patient is reassured and comforted. d. The patient is trying to manipulate the nurse using nonverbal techniques.

ANS: A The most likely answer is that the patient's behavior is culturally influenced. Hispanic women frequently touch women they consider to be their friends. Although the other options are possible, they are less likely.

A nurse uses Maslow's hierarchy of needs to plan care for a patient with mental illness. Which problem will receive priority? The patient: a. refuses to eat or bathe. b. reports feelings of alienation from family. c. is reluctant to participate in unit social activities. d. is unaware of medication action and side effects.

ANS: A The need for food and hygiene are physiological and therefore take priority over psychological or meta-needs in care planning.

As a nurse discharges a patient, the patient gives the nurse a card of appreciation made in an arts and crafts group. What is the nurse's best action? a. Recognize the effectiveness of the relationship and patient's thoughtfulness. Accept the card. b. Inform the patient that accepting gifts violates policies of the facility. Decline the card. c. Acknowledge the patient's transition through the termination phase but decline the card. d. Accept the card and invite the patient to return to participate in other arts and crafts groups.

ANS: A The nurse must consider the meaning, timing, and value of the gift. In this instance, the nurse should accept the patient's expression of gratitude. See relationship to audience response question.

A patient with acute depression states, "God is punishing me for my past sins." What is the nurse's most therapeutic response? a. "You sound very upset about this." b. "God always forgives us for our sins." c. "Why do you think you are being punished?" d. "If you feel this way, you should talk to your minister."

ANS: A The nurse reflects the patient's comment, a therapeutic technique to encourage sharing for perceptions and feelings. The incorrect responses reflect probing, closed-ended comments, and giving advice, all of which are non-therapeutic.

A patient diagnosed with schizophrenia has been stable for 2 months. Today the patient's spouse calls the nurse to report the patient has not taken prescribed medication and is having disorganized thinking. The patient forgot to refill the prescription. The nurse arranges a refill. Select the best outcome to add to the plan of care. a. The patient's spouse will mark dates for prescription refills on the family calendar. b. The nurse will obtain prescription refills every 90 days and deliver to the patient. c. The patient will call the nurse weekly to discuss medication-related issues. d. The patient will report to the clinic for medication follow-up every week.

ANS: A The nurse should use the patient's support system to meet patient needs whenever possible. Delivery of medication by the nurse should be unnecessary for the nurse to do if patient or a significant other can be responsible. The patient may not need more intensive follow-up as long as medication is taken as prescribed.

At what point in the nurse-patient relationship should a nurse plan to first address termination? a. During the orientation phase b. At the end of the working phase c. Near the beginning of the termination phase d. When the patient initially brings up the topic

ANS: A The patient has a right to know the conditions of the nurse-patient relationship. If the relationship is to be time-limited, the patient should be informed of the number of sessions. If it is open-ended, the termination date will not be known at the outset, and the patient should know that the issue will be negotiated at a later date. The nurse is responsible for bringing up the topic of termination early in the relationship, usually during the orientation phase.

A nurse performed these actions while caring for patients in an inpatient psychiatric setting. Which action violated patients' rights? a. Prohibited a patient from using the telephone b. In patient's presence, opened a package mailed to patient c. Remained within arm's length of patient with homicidal ideation d. Permitted a patient with psychosis to refuse oral psychotropic medication

ANS: A The patient has a right to use the telephone. The patient should be protected against possible harm to self or others. Patients have rights to send and receive mail and be present during package inspection. Patients have rights to refuse treatment.

A nurse surveys medical records. Which finding signals a violation of patients' rights? a. A patient was not allowed to have visitors. b. A patient's belongings were searched at admission. c. A patient with suicidal ideation was placed on continuous observation. d. Physical restraint was used after a patient was assaultive toward a staff member.

ANS: A The patient has the right to have visitors. Inspecting patients' belongings is a safety measure. Patients have the right to a safe environment, including the right to be protected against impulses to harm self.

12. A patient is experiencing delusions of persecution about being poisoned. The patient has refused all hospital meals for 3 days. Which intervention is most likely to be acceptable to the patient? a. Allowing the patient supervised access to food vending machines b. Allowing the patient to phone a local restaurant to deliver meals c. Offering to taste each portion on the tray for the patient d. Providing tube feedings or total parenteral nutrition

ANS: A The patient who is delusional about food being poisoned is likely to believe restaurant food might still be poisoned and to say that the staff member tasting the food has taken an antidote to the poison before tasting. Attempts to tube feed or give nutrition intravenously are seen as aggressive and usually promote violence. Patients perceive foods in sealed containers, packages, or natural shells as being safer.

When a nurse assesses an older adult patient, answers seem vague or unrelated to the questions. The patient also leans forward and frowns, listening intently to the nurse. An appropriate question for the nurse to ask would be: a. "Are you having difficulty hearing when I speak?" b. "How can I make this assessment interview easier for you?" c. "I notice you are frowning. Are you feeling annoyed with me?" d. "You're having trouble focusing on what I'm saying. What is distracting you?"

ANS: A The patient's behaviors may indicate difficulty hearing. Identifying any physical need the patient may have at the onset of the interview and making accommodations are important considerations. By asking if the patient is annoyed, the nurse is jumping to conclusions. Asking how to make the interview easier for the patient may not elicit a concrete answer. Asking about distractions is a way of asking about auditory hallucinations, which is not appropriate because the nurse has observed that the patient seems to be listening intently.

In the majority culture of the United States, which individual has the greatest risk to be labeled mentally ill? One who: a. describes hearing God's voice speaking. b. is usually pessimistic but strives to meet personal goals. c. is wealthy and gives away $20 bills to needy individuals. d. always has an optimistic viewpoint about life and having own needs met.

ANS: A The question asks about risk. Hearing voices is generally associated with mental illness, but in charismatic religious groups, hearing the voice of God or a prophet is a desirable event. Cultural norms vary, which makes it more difficult to make an accurate diagnosis. The individuals described in the other options are less likely to be labeled mentally ill.

Which principle has the highest priority when addressing a behavioral crisis in an inpatient setting? a. Resolve the crisis with the least restrictive intervention possible. b. Swift intervention is justified to maintain the integrity of a therapeutic milieu. c. Rights of an individual patient are superseded by the rights of the majority of patients. d. Patients should have opportunities to regain control without intervention if the safety of others is not compromised.

ANS: A The rule of using the least restrictive treatment or intervention possible to achieve the desired outcome is the patient's legal right. Planned interventions are nearly always preferable. Intervention may be necessary when the patient threatens harm to self.

Consider this comment from a therapist: "The patient is homosexual but has kept this preference secret. Severe anxiety and depression occur when the patient anticipates family reactions to this sexual orientation." Which perspective is evident in the speaker? a. Theory of interpersonal relationships b. Classical conditioning theory c. Psychosexual theory d. Behaviorism theory

ANS: A The theory of interpersonal relationships recognizes the anxiety and depression as resulting from unmet interpersonal security needs. Behaviorism and classical conditioning theories do not apply. A psychosexual formulation would focus on uncovering unconscious material that relates to the patient problem.

Which scenario best depicts a behavioral crisis? A patient is: a. waving fists, cursing, and shouting threats at a nurse. b. curled up in a corner of the bathroom, wrapped in a towel. c. crying hysterically after receiving a phone call from a family member. d. performing push-ups in the middle of the hall, forcing others to walk around.

ANS: A This behavior constitutes a behavioral crisis because the patient is threatening harm to another individual. Intervention is called for to defuse the situation. The other options speak of behaviors that may require intervention of a less urgent nature because the patients in question are not threatening harm to self or others.

4. When a patient diagnosed with schizophrenia was discharged 6 months ago, haloperidol (Haldol) was prescribed. The patient now says, "I stopped taking those pills. They made me feel like a robot." What are common side effects the nurse should validate with the patient? a. Sedation and muscle stiffness b. Sweating, nausea, and diarrhea c. Mild fever, sore throat, and skin rash d. Headache, watery eyes, and runny nose

ANS: A Typical antipsychotic drugs often produce sedation and extrapyramidal side effects such as stiffness and gait disturbance, effects the patient might describe as making him or her feel like a "robot." The side effects mentioned in the other options are usually not associated with typical antipsychotic therapy or would not have the effect described by the patient.

A nurse says, "I am the only one who truly understands this patient. Other staff members are too critical." The nurse's statement indicates: a. boundary blurring. b. sexual harassment. c. positive regard. d. advocacy.

ANS: A When the role of the nurse and the role of the patient shift, boundary blurring may arise. In this situation the nurse is becoming over-involved with the patient as a probable result of unrecognized countertransference. When boundary issues occur, the need for supervision exists. The situation does not describe sexual harassment. Data are not present to suggest positive regard or advocacy.

22. What assessment findings mark the prodromal stage of schizophrenia? a. Withdrawal, misinterpreting, poor concentration, and preoccupation with religion b. Auditory hallucinations, ideas of reference, thought insertion, and broadcasting c. Stereotyped behavior, echopraxia, echolalia, and waxy flexibility d. Loose associations, concrete thinking, and echolalia neologisms

ANS: A Withdrawal, misinterpreting, poor concentration, and preoccupation with religion are prodromal symptoms, the symptoms that are present before the development of florid symptoms. The incorrect options each list the positive symptoms of schizophrenia that might be apparent during the acute stage of the illness.

A consumer at a rehabilitative psychosocial program says to the nurse, "People are not cleaning up behind themselves in the bathrooms. The building is dirty and cluttered." How should the nurse respond? a. Encourage the consumer to discuss it at a meeting with everyone. b. Hire a professional cleaning service to clean the restrooms. c. Address the complaint at the next staff meeting. d. Tell the consumer, "That's not my problem."

ANS: A Consumer-run programs range from informal "clubhouses," which offer socialization and recreation, to competitive businesses, such as snack bars or janitorial services, which provide needed services and consumer employment while encouraging independence and building vocational skills. Consumers engage in problem solving under the leadership of staff. See related audience response question.

A patient diagnosed with a serious mental illness lives independently and attends a psychosocial rehabilitation program. The patient presents at the emergency department seeking hospitalization. The patient has no acute symptoms but says, "I have no money to pay my rent or refill my prescription." Select the nurse's best action. a. Involve the patient's case manager to provide crisis intervention. b. Send the patient to a homeless shelter until housing can be arranged. c. Arrange for a short in-patient admission and begin discharge planning. d. Explain that one must have active psychiatric symptoms to be admitted.

ANS: A Impaired stress tolerance and problem-solving abilities can cause persons with SMI to experience relatively minor stressors as crises. This patient has run out of money, and this has overwhelmed her ability to cope, resulting in a crisis for which crisis intervention would be an appropriate response. Inpatient care is not clinically indicated nor is the patient homeless (although she may fear she is). Telling the patient that she is not symptomatic enough to be admitted may prompt malingering.

Which assessment finding most clearly indicates that a patient may be experiencing a mental illness? The patient: a. reports occasional sleeplessness and anxiety. b. reports a consistently sad, discouraged, and hopeless mood. c. is able to describe the difference between "as if" and "for real." d. perceives difficulty making a decision about whether to change jobs.

ANS: B The correct response describes a mood alteration, which reflects mental illness. The distracters describe behaviors that are mentally healthy or within the usual scope of human experience.

A nurse's neighbor says, "My sister has been diagnosed with bipolar disorder but will not take her medication. I have tried to help her for over 20 years, but it seems like everything I do fails. Do you have any suggestions?" Select the nurse's best response. a. "The National Alliance on Mental Illness offers a family education series that you might find helpful." b. "Since your sister is noncompliant, perhaps it's time for her to be changed to injectable medication." c. "You have done all you can. Now it's time to put yourself first and move on with your life." d. "You cannot help her. Would it be better for you to discontinue your relationship?"

ANS: A The National Alliance on Mental Illness (NAMI) offers a family education series that assists with the stress caregivers and other family members often experience. The nurse should not give advice about injectable medication or encourage the family member to give up on the patient.

5. Which hallucination necessitates the nurse to implement safety measures? The patient says, a. "I hear angels playing harps." b. "The voices say everyone is trying to kill me." c. "My dead father tells me I am a good person." d. "The voices talk only at night when I'm trying to sleep."

ANS: B The correct response indicates the patient is experiencing paranoia. Paranoia often leads to fearfulness, and the patient may attempt to strike out at others to protect self. The distracters are comforting hallucinations or do not indicate paranoia.

2. A patient diagnosed with schizophrenia was hospitalized after arguing with co-workers and threatening to harm them. The patient is aloof, suspicious, and says, "Two staff members I saw talking were plotting to kill me." Based on data gathered at this point, which nursing diagnoses relate? Select all that apply. a. Risk for other-directed violence b. Disturbed thought processes c. Risk for loneliness d. Spiritual distress e. Social isolation

ANS: A, B Delusions of persecution and ideas of reference support the nursing diagnosis of disturbed thought processes. Risk for other-directed violence is substantiated by the patient's feeling endangered by persecutors. Fearful individuals may strike out at perceived persecutors or attempt self-harm to get away from persecutors. Data are not present to support the other diagnoses.

An experienced nurse says to a new graduate, "When you've practiced as long as I have, you instantly know how to take care of psychotic patients." What information should the new graduate consider when analyzing this comment? Select all that apply. a. The experienced nurse may have lost sight of patients' individuality, which may compromise the integrity of practice. b. New research findings should be integrated continuously into a nurse's practice to provide the most effective care. c. Experience provides mental health nurses with the essential tools and skills needed for effective professional practice. d. Experienced psychiatric nurses have learned the best ways to care for mentally ill patients through trial and error. e. An intuitive sense of patients' needs guides effective psychiatric nurses.

ANS: A, B Evidence-based practice involves using research findings and standards of care to provide the most effective nursing care. Evidence is continuously emerging, so nurses cannot rely solely on experience. The effective nurse also maintains respect for each patient as an individual. Overgeneralization compromises that perspective. Intuition and trial and error are unsystematic approaches to care.

A nurse ends a relationship with a patient. Which actions by the nurse should be included in the termination phase? Select all that apply. a. Focus dialogues with the patient on problems that may occur in the future. b. Help the patient express feelings about the relationship with the nurse. c. Help the patient prioritize and modify socially unacceptable behaviors. d. Reinforce expectations regarding the parameters of the relationship. e. Help the patient to identify strengths, limitations, and problems.

ANS: A, B The correct actions are part of the termination phase. The other actions would be used in the working and orientation phases.

A nurse performing an assessment interview for a patient with a substance use disorder decides to use a standardized rating scale. Which scales are appropriate? Select all that apply. a. Addiction Severity Index (ASI) b. Brief Drug Abuse Screen Test (B-DAST) c. Abnormal Involuntary Movement Scale (AIMS) d. Cognitive Capacity Screening Examination (CCSE) e. Recovery Attitude and Treatment Evaluator (RAATE)

ANS: A, B, E Standardized scales are useful for obtaining data about substance use disorders. The ASI, B-DAST, and RAATE are scales related to substance abuse. AIMS assesses involuntary movements associated with anti-psychotic medications. The CCSE assesses cognitive function.

Which benefits are most associated with use of telehealth technologies? Select all that apply. a. Cost savings for patients b. Maximize care management c. Access to services for patients in rural areas d. Prompt reimbursement by third party payers e. Rapid development of trusting relationships with patients

ANS: A, B, C Telehealth has shown it can maximize health and improve disease management skills and confidence with the disease process. Many rural parents have felt disconnected from services; telehealth technologies can solve those problems. Although telehealth's improved health outcomes regularly show cost savings for payers, one significant barrier is the current lack of reimbursement for remote patient monitoring by third party payers. Telehealth technologies have not shown rapid development of trusting relationships.

Select all that apply. An adult patient tells the case manager, "I don't have bipolar disorder anymore, so I don't need medicine. After I was in the hospital last year, you helped me get an apartment and disability checks. Now I'm bored and don't have any friends." Where should the nurse refer the patient? a. Psychoeducational classes b. Vocational rehabilitation c. Social skills training d. A homeless shelter e. Crisis intervention

ANS: A, B, C The patient does not understand the illness and need for adherence to the medication regimen. Psychoeducation for the patient (and family) can address this lack of knowledge. The patient, who considers himself friendless, could also profit from social skills training to improve the quality of interpersonal relationships. Many patients with serious mental illness have such poor communication skills that others are uncomfortable interacting with them. Interactional skills can be effectively taught by breaking the skill down into smaller verbal and nonverbal components. Work gives meaning and purpose to life, so vocational rehabilitation can assist with this aspect of care. The nurse case manager will function in the role of crisis stabilizer, so no related referral is needed. The patient presently has a home and does not require a homeless shelter.

1. A nurse can best address factors of critical importance to successful community treatment by including making assessments relative to: (Select all that apply.) a. housing adequacy. b. family and support systems. c. income adequacy and stability. d. early psychosocial development. e. substance abuse history and current use.

ANS: A, B, C, E Early psychosocial developmental history is less relevant to successful outcomes in the community than the assessments listed in the other options. If a patient is homeless or fears homelessness, focusing on other treatment issues is impossible. Sufficient income for basic needs and medication is necessary. Adequate support is a requisite to community placement. Substance abuse undermines medication effectiveness and interferes with community adjustment.

1. A patient states, "I'm starting cognitive-behavioral therapy. What can I expect from the sessions?" Which responses by the nurse would be appropriate? Select all that apply. a. "The therapist will be active and questioning." b. "You will be given some homework assignments." c. "The therapist will ask you to describe your dreams." d. "The therapist will help you look at your ideas and beliefs about yourself." e. "The goal is to increase subjectivity about thoughts that govern your behavior."

ANS: A, B, D Cognitive therapists are active rather than passive during therapy sessions because they help patients reality-test their thinking. Homework assignments are given and completed outside the therapy sessions. Homework is usually discussed at the next therapy session. The goal of cognitive therapy is to assist the patient in identifying inaccurate cognitions and in reality- testing and formulating new, accurate cognitions. One distracter applies to psychoanalysis. Increasing subjectivity is not desirable.

Which statements by patients diagnosed with a serious mental illness best demonstrate that the case manager has established an effective long-term relationship? "My case manager: (select all that apply) a. talks in language I can understand." b. helps me keep track of my medication." c. gives me little gifts from time to time." d. looks at me as a whole person with many needs." e. lets me do whatever I choose without interfering."

ANS: A, B, D Each correct answer is an example of appropriate nursing foci: communicating at a level understandable to the patient, providing medication supervision, and using holistic principles to guide care. The distracters violate relationship boundaries or suggest a laissez faire attitude on the part of the nurse.

A psychiatric nurse discusses rules of the therapeutic milieu and patients' rights with a newly admitted patient. Which rights should be included? (Select all that apply.) The right to: a. have visitors b. confidentiality c. a private room d. complain about inadequate care e. select the nurse assigned to their care

ANS: A, B, D Patients' rights should be discussed shortly after admission. Patients have rights related to receiving/refusing visitors, privacy, filing complaints about inadequate care, and accepting/refusing treatments (including medications). Patients do not have a right to a private room or selecting which nurse will provide care.

A person in the community asks, "People with mental illnesses went to state hospitals in earlier times. Why has that changed?" Select the nurse's accurate responses. Select all that apply. a. "Science has made significant improvements in drugs for mental illness, so now many persons may live in their communities." b. "There's now a better selection of less restrictive treatment options available in communities to care for people with mental illness." c. "National rates of mental illness have declined significantly. There actually is not a need for state institutions anymore." d. "Most psychiatric institutions were closed because of serious violations of patients' rights and unsafe conditions." e. "Federal legislation and payment for treatment of mental illness has shifted the focus to community rather than institutional settings."

ANS: A, B, E The community is a less restrictive alternative than hospitals for treatment of persons with mental illness. Funding for treatment of mental illness remains largely inadequate but now focuses on community rather than institutional care. Antipsychotic medications improve more symptoms of mental illness; hence, management of psychiatric disorders has improved. Rates of mental illness have increased, not decreased. Hospitals were closed because funding shifted to the community. Conditions in institutions have improved.

Select all that apply. Which statements most clearly indicate the speaker views mental illness with stigma? a. "We are all a little bit crazy." b. "If people with mental illness would go to church, their problems would be solved." c. "Many mental illnesses are genetically transmitted. It's no one's fault that the illness occurs." d. "Anyone can have a mental illness. War or natural disasters can be too stressful for healthy people." e. "People with mental illness are lazy. They get government disability checks instead of working."

ANS: A, B, E Stigma is represented by judgmental remarks that discount the reality and validity of mental illness. It is evidenced in stereotypical statements, by oversimplification, and by multiple other messages of guilt or shame. See related audience response question.

Select all that apply. A patient diagnosed with serious mental illness was living successfully in a group home but wanted an apartment. The prospective landlord said, "People like you have trouble getting along and paying their rent." The patient and nurse meet for a problem-solving session. Which options should the nurse endorse? a. Coach the patient in ways to control symptoms effectively. b. Seek out landlords less affected by the stigma associated with mental illness. c. Threaten the landlord with legal action because of the discriminatory actions. d. Encourage the patient to remain in the group home until the illness is less obvious. e. Suggest that the patient list a false current address in the rental application. f. Have the case manager meet with the landlord to provide education about mental illness.

ANS: A, B, F Managing symptoms so that they are less obvious or socially disruptive can reduce negative reactions and reduce rejection due to stigma. Seeking a more receptive landlord might be the most expeditious route to housing for this patient. Educating the landlord to reduce stigma might make him more receptive and give the case manager an opportunity to address some of his concerns (e.g., the case manager could arrange a payee to assure that the rent is paid each month). However, threatening a lawsuit would increase the landlord's defensiveness and would likely be a long and expensive undertaking. Delaying the patient's efforts to become more independent is not clinically necessary according to the data noted here; the problem is the landlord's bias and response, not the patient's illness. It would be unethical to encourage falsification and poor role modeling to do so; further, if falsification is discovered, it could permit the landlord to refuse or cancel her lease. See related audience response question.

A nurse is interacting with patients in a psychiatric unit. Which statements reflect use of therapeutic communication? Select all that apply. a. "Tell me more about that situation." b. "Let's talk about something else." c. "I notice you are pacing a lot." d. "I'll stay with you a while." e. "Why did you do that?"

ANS: A, C, D The correct responses demonstrate use of the therapeutic techniques making an observation and showing empathy. The incorrect responses demonstrate changing the subject and probing, which are non-therapeutic techniques.

The health care team at an inpatient psychiatric facility drafts these criteria for admission. Which criteria should be included in the final version of the admission policy? Select all that apply. a. Clear risk of danger to self or others b. Adjustment needed for doses of psychotropic medication c. Detoxification from long-term heavy alcohol consumption needed d. Respite for caregivers of persons with serious and persistent mental illness e. Failure of community-based treatment, demonstrating need for intensive treatment

ANS: A, C, E Medication doses can be adjusted on an outpatient basis. The goal of caregiver respite can be accomplished without hospitalizing the patient. The other options are acceptable, evidence-based criteria for admission of a patient to an inpatient service.

A patient diagnosed with schizophrenia lives in the community. On a home visit, the community psychiatric nurse case manager learns that the patient: · wants to attend an activity group at the mental health outreach center. · is worried about being able to pay for the therapy. · does not know how to get from home to the outreach center. · has an appointment to have blood work at the same time an activity group meets. · wants to attend services at a church that is a half-mile from the patient's home. Which tasks are part of the role of a community mental health nurse? Select all that apply. a. Rearranging conflicting care appointments b. Negotiating the cost of therapy for the patient c. Arranging transportation to the outreach center d. Accompanying the patient to church services weekly e. Monitoring to ensure the patient's basic needs are met

ANS: A, C, E The correct answers reflect the coordinating role of the community psychiatric nurse case manager. Negotiating the cost of therapy and accompanying the patient to church services are interventions the nurse would not be expected to undertake. The patient can walk to the church services; the nurse can provide encouragement.

Which comments by an elderly person best indicate successful completion of the developmental task? Select all that apply. a. "I am proud of my children's successes in life." b. "I should have given to community charities more often." c. "My relationship with my father made life more difficult for me." d. "My experiences in the war helped me appreciate the meaning of life." e. "I often wonder what would have happened if I had chosen a different career."

ANS: A, D The developmental crisis for an elderly person relates to integrity versus despair. Pride in one's offspring indicates a sense of fulfillment. Recognition of the wisdom gained from difficult experiences (such as being in a war) indicates a sense of integrity. Blaming and regret indicate despair and unsuccessful resolution of the crisis.

Which findings are signs of a person who is mentally healthy? Select all that apply. a. Says, "I have some weaknesses, but I feel I'm important to my family and friends." b. Adheres strictly to religious beliefs of parents and family of origin. c. Spends all holidays alone watching old movies on television. d. Considers past experiences when deciding about the future. e. Experiences feelings of conflict related to changing jobs.

ANS: A, D, E Mental health is a state of well-being in which each individual is able to realize his or her own potential, cope with the normal stresses of life, work productively, and make a contribution to the community. Mental health provides people with the capacity for rational thinking, communication skills, learning, emotional growth, resilience, and self-esteem.

Which comments by an adult best indicate self-actualization? Select all that apply. a. "I am content with a good book." b. "I often wonder if I chose the right career." c. "Sometimes I think about how my parents would have handled problems." d. "It's important for our country to provide basic health care services for everyone." e. "When I was lost at sea for 2 days, I gained an understanding of what is important."

ANS: A, D, E Self-actualized persons enjoy privacy, have a sense of democracy, and show positive outcomes associated with peak experiences. Self-doubt, defensiveness, and blaming are not consistent with self-actualization.

1. A nurse at the mental health clinic plans a series of psychoeducational groups for persons newly diagnosed with schizophrenia. Which two topics take priority? a. "The importance of taking your medication correctly" b. "How to complete an application for employment" c. "How to dress when attending community events" d. "How to give and receive compliments" e. "Ways to quit smoking"

ANS: A, E Stabilization is maximized by adherence to the antipsychotic medication regimen. Because so many persons with schizophrenia smoke cigarettes, this topic relates directly to the patients' physiological well-being. The other topics are also important but are not priority topics.

A novice psychiatric nurse has a parent with bipolar disorder. This nurse angrily recalls feelings of embarrassment about the parent's behavior in the community. Select the best ways for this nurse to cope with these feelings.Select all that apply. a. Seek ways to use the understanding gained from childhood to help patients cope with their own illnesses. b. Recognize that these feelings are unhealthy. The nurse should try to suppress them when working with patients. c. Recognize that psychiatric nursing is not an appropriate career choice. Explore other nursing specialties. d. The nurse should begin new patient relationships by saying, "My own parent had mental illness, so I accept it without stigma." e. Recognize that the feelings may add sensitivity to the nurse's practice, but supervision is important.

ANS: A, E The nurse needs support to explore these feelings. An experienced psychiatric nurse is a resource that may be helpful. The knowledge and experience gained from the nurse's relationship with a mentally ill parent may contribute sensitivity to compassionate practice. Self-disclosure and suppression are not adaptive coping strategies. The nurse should not give up on this area of practice without first seeking ways to cope with the memories.

Select all that apply. The nurse manager of a mental health center wants to improve medication adherence among the seriously mentally ill persons treated there. Which interventions are likely to help achieve this goal? a. Maintain stable and consistent staff. b. Increase the length of medication education groups. c. Stress that without treatment, illnesses will worsen. d. Prescribe drugs in smaller but more frequent dosages. e. Make it easier to access prescribers and pay for drugs. f. Require adherence in order to participate in programming.

ANS: A, E Trust in one's providers is a key factor in treatment adherence, and mentally ill persons can sometimes take a very long time to develop such trust; therefore, interventions which stabilize staffing allow patients to have more time with staff to develop these bonds. Ready access to prescribers allows medicine-related concerns to be addressed quickly, reducing obstacles to adherence such as side effects or ineffective dosages. Medication costs can be obstacles to adherence as well. Many SMI patients have anosognosia and do not adhere to treatment because they believe they are not ill, so telling them nonadherence will worsen an illness they do not believe they have is unlikely to be helpful. Increasing medication education is helpful only when the cause of nonadherence is a knowledge deficit. Other issues that reduce adherence, particularly anosognosia and side effects, are seldom helped by longer medication education. Requiring medication adherence to participate in other programs is coercive and unethical. Smaller, more frequent doses do not reduce side effects and make the regimen more difficult for the patient to remember.

Which characteristic would be more applicable to a community mental health nurse than to a nurse working in an operating room? a. Kindness b. Autonomy c. Compassion d. Professionalism

ANS: B A community mental health nurse often works autonomously. Kindness, compassion, and professionalism apply to both nurses.

A nurse introduces the matter of a contract during the first session with a new patient because contracts: a. specify what the nurse will do for the patient. b. spell out the participation and responsibilities of each party. c. indicate the feeling tone established between the participants. d. are binding and prevent either party from prematurely ending the relationship.

ANS: B A contract emphasizes that the nurse works with the patient rather than doing something for the patient. "Working with" is a process that suggests each party is expected to participate and share responsibility for outcomes. Contracts do not, however, stipulate roles or feeling tone, and premature termination is forbidden.

Which patient statement would lead the nurse to suspect unsuccessful completion of the developmental task of infancy? a. "I have very warm and close friendships." b. "I'm afraid to allow anyone to really get to know me." c. "I'm always absolutely right, so don't bother saying more." d. "I'm ashamed that I didn't do things correctly in the first place."

ANS: B According to Erikson, the developmental task of infancy is the development of trust. The correct response is the only statement clearly showing lack of ability to trust others. Warm, close relationships suggest the developmental task of infancy was successfully completed; rigidity and self-absorption are reflected in the belief one is always right; and shame for past actions suggests failure to resolve the crisis of initiative versus guilt.

Which level of prevention activities would a nurse in an emergency department employ most often? a. Primary b. Secondary c. Tertiary

ANS: B An emergency department nurse would generally see patients in crisis or with acute illness, so secondary prevention is used. Primary prevention involves preventing a health problem from developing, and tertiary prevention applies to rehabilitative activities.

Complete this analogy. NANDA: clinical judgment: NIC: _________________ a. patient outcomes b. nursing actions c. diagnosis d. symptoms

ANS: B Analogies show parallel relationships. NANDA, the North American Nursing Diagnosis Association, identifies diagnostic statements regarding human responses to actual or potential health problems. These statements represent clinical judgments. NIC (Nursing Interventions Classification) identifies actions provided by nurses that enhance patient outcomes. Nursing care activities may be direct or indirect.

A nurse wants to find a description of diagnostic criteria for anxiety disorders. Which resource would have the most complete information? a. Nursing Outcomes Classification (NOC) b. Diagnostic and Statistical Manual of Mental Disorders (DSM-5) c. The ANA's Psychiatric-Mental Health Nursing Scope and Standards of Practice d. International Statistical Classification of Diseases and Related Health Problems (ICD-10)

ANS: B The DSM-5 details the diagnostic criteria for psychiatric clinical conditions. The other references are good resources but do not define the diagnostic criteria.

A nurse asks a patient, "If you had fever and vomiting for 3 days, what would you do?" Which aspect of the mental status examination is the nurse assessing? a. Behavior b. Cognition c. Affect and mood d. Perceptual disturbances

ANS: B Assessing cognition involves determining a patient's judgment and decision making. In this case, the nurse would expect a response of "Call my doctor" if the patient's cognition and judgment are intact. If the patient responds, "I would stop eating" or "I would just wait and see what happened," the nurse would conclude that judgment is impaired. The other options refer to other aspects of the examination.

Before assessing a new patient, a nurse is told by another health care worker, "I know that patient. No matter how hard we work, there isn't much improvement by the time of discharge." The nurse's responsibility is to: a. document the other worker's assessment of the patient. b. assess the patient based on data collected from all sources. c. validate the worker's impression by contacting the patient's significant other. d. discuss the worker's impression with the patient during the assessment interview.

ANS: B Assessment should include data obtained from both the primary and reliable secondary sources. The nurse, bearing in mind the possible effects of counter-transference, should evaluate biased assessments by others as objectively as possible.

A patient says, "All my life I've been surrounded by stupidity. Everything I buy breaks because the entire American workforce is incompetent." This patient is experiencing a: a. self-esteem deficit. b. cognitive distortion. c. deficit in motivation. d. deficit in love and belonging.

ANS: B Automatic thoughts, or cognitive distortions, are irrational and lead to false assumptions and misinterpretations. See related audience response question.

A patient says, "I always feel good when I wear a size 2 petite." Which type of cognitive distortion is evident? a. Disqualifying the positive b. Overgeneralization c. Catastrophizing d. Personalization

ANS: B Automatic thoughts, or cognitive distortions, are irrational and lead to false assumptions and misinterpretations. The stem offers an example of overgeneralization. See related audience response question.

6. A patient's care plan includes monitoring for auditory hallucinations. Which assessment findings suggest the patient may be hallucinating? a. Detachment and overconfidence b. Darting eyes, tilted head, mumbling to self c. Euphoric mood, hyperactivity, distractibility d. Foot tapping and repeatedly writing the same phrase

ANS: B Clues to hallucinations include eyes looking around the room as though to find the speaker, tilting the head to one side as though listening intently, and grimacing, mumbling, or talking aloud as though responding conversationally to someone.

A nurse assesses a confused older adult. The nurse experiences sadness and reflects, "The patient is like one of my grandparents...so helpless." Which response is the nurse demonstrating? a. Transference b. Countertransference c. Catastrophic reaction d. Defensive coping reaction

ANS: B Countertransference is the nurse's transference or response to a patient that is based on the nurse's unconscious needs, conflicts, problems, or view of the world. See relationship to audience response question.

An advanced practice nurse observes a novice nurse expressing irritability regarding a patient with a long history of alcoholism and suspects the new nurse is experiencing countertransference. Which comment by the new nurse confirms this suspicion? a. "This patient continues to deny problems resulting from drinking." b. "My parents were alcoholics and often neglected our family." c. "The patient cannot identify any goals for improvement." d. "The patient said I have many traits like her mother."

ANS: B Countertransference occurs when the nurse unconsciously and inappropriately displaces onto the patient feelings and behaviors related to significant figures in the nurse's past. In this instance, the new nurse's irritability stems from relationships with parents. The distracters indicate transference or accurate analysis of the patient's behavior.

A patient presents to the emergency department with mixed psychiatric symptoms. The admission nurse suspects the symptoms may be the result of a medical problem. Lab results show elevated BUN (blood urea nitrogen) and creatinine. What is the nurse's next best action? a. Report the findings to the health care provider. b. Assess the patient for a history of renal problems. c. Assess the patient's family history for cardiac problems. d. Arrange for the patient's hospitalization on the psychiatric unit.

ANS: B Elevated BUN (blood urea nitrogen) and creatinine suggest renal problems. Renal dysfunction can often imitate psychiatric disorders. The nurse should further assess the patient's history for renal problems and then share the findings with the health care provider.

A nurse wants to demonstrate genuineness with a patient diagnosed with schizophrenia. The nurse should: a. restate what the patient says. b. use congruent communication strategies. c. use self-revelation in patient interactions. d. consistently interpret the patient's behaviors.

ANS: B Genuineness is a desirable characteristic involving awareness of one's own feelings as they arise and the ability to communicate them when appropriate. The incorrect options are undesirable in a therapeutic relationship.

A black patient says to a white nurse, "There's no sense talking. You wouldn't understand because you live in a white world." The nurse's best action would be to: a. explain, "Yes, I do understand. Everyone goes through the same experiences." b. say, "Please give an example of something you think I wouldn't understand." c. reassure the patient that nurses interact with people from all cultures. d. change the subject to one that is less emotionally disturbing.

ANS: B Having the patient speak in specifics rather than globally will help the nurse understand the patient's perspective. This approach will help the nurse engage the patient. Reassurance and changing the subject are not therapeutic techniques.

A newly admitted patient diagnosed with schizophrenia is hypervigilant and constantly scans the environment. The patient states, "I saw two doctors talking in the hall. They were plotting to kill me." The nurse may correctly assess this behavior as: a. echolalia. b. an idea of reference. c. a delusion of infidelity. d. an auditory hallucination.

ANS: B Ideas of reference are misinterpretations of the verbalizations or actions of others that give special personal meanings to these behaviors; for example, when seeing two people talking, the individual assumes they are talking about him or her. The other terms do not correspond with the scenario.

1. A staff nurse completes orientation to a psychiatric unit. This nurse may expect an advanced practice nurse to perform which additional intervention? a. Conduct mental health assessments. b. Prescribe psychotropic medication. c. Establish therapeutic relationships. d. Individualize nursing care plans.

ANS: B In most states, prescriptive privileges are granted to master's-prepared nurse practitioners who have taken special courses on prescribing medication. The nurse prepared at the basic level is permitted to perform mental health assessments, establish relationships, and provide individualized care planning.

A patient diagnosed with schizophrenia tells the nurse, "The CIA is monitoring us through the fluorescent lights in this room. Be careful what you say." Which response by the nurse would be most therapeutic? a. "Let's talk about something other than the CIA." b. "It sounds like you're concerned about your privacy." c. "The CIA is prohibited from operating in health care facilities." d. "You have lost touch with reality, which is a symptom of your illness."

ANS: B It is important not to challenge the patient's beliefs, even if they are unrealistic. Challenging undermines the patient's trust in the nurse. The nurse should try to understand the underlying feelings or thoughts the patient's message conveys. The correct response uses the therapeutic technique of reflection. The other comments are non-therapeutic. Asking to talk about something other than the concern at hand is changing the subject. Saying that the CIA is prohibited from operating in health care facilities gives false reassurance. Stating that the patient has lost touch with reality is truthful, but uncompassionate.

24. A patient diagnosed with schizophrenia begins a new prescription for lurasidone HCL (Latuda). The patient is 5'6" and currently weighs 204 lbs. Which topic is most important for the nurse to include in the teaching plan related to this medication? a. How to recognize tardive dyskinesia b. Weight management strategies c. Ways to manage constipation d. Sleep hygiene measures

ANS: B Lurasidone HCL (Latuda) is a second-generation antipsychotic medication. The incidence of weight gain, diabetes, and high cholesterol is high with this medication. The patient is overweight now, so weight management will be especially important. The incidence of tardive dyskinesia is low with second-generation antipsychotic medications. Constipation may occur, but it is less important than weight management. This drug usually produces drowsiness.

A nurse encounters an unfamiliar psychiatric disorder on a new patient's admission form. Which resource should the nurse consult to determine criteria used to establish this diagnosis? a. International Statistical Classification of Diseases and Related Health Problems (ICD-10) b. Diagnostic and Statistical Manual of Mental Disorders (DSM-5) c. A behavioral health reference manual d. Wikipedia

ANS: B The DSM-5 gives the criteria used to diagnose each mental disorder. The distracters may not contain diagnostic criteria for a psychiatric illness.

37. A patient insistently states, "I can decipher codes of DNA just by looking at someone." Which problem is evident? a. Visual hallucinations b. Magical thinking c. Idea of reference d. Thought insertion

ANS: B Magical thinking is evident in the patient's appraisal of his own abilities. There is no evidence of the distracters.

The patient says, "My marriage is just great. My spouse and I always agree." The nurse observes the patient's foot moving continuously as the patient twirls a shirt button. The conclusion the nurse can draw is that the patient's communication is: a. clear. b. mixed. c. precise. d. inadequate.

ANS: B Mixed messages involve the transmission of conflicting or incongruent messages by the speaker. The patient's verbal message that all was well in the relationship was modified by the nonverbal behaviors denoting anxiety. Data are not present to support the choice of the verbal message being clear, explicit, or inadequate.

A nurse receives these three phone calls regarding a newly admitted patient. · The psychiatrist wants to complete an initial assessment. · An internist wants to perform a physical examination. · The patient's attorney wants an appointment with the patient. The nurse schedules the activities for the patient. Which role has the nurse fulfilled? a. Advocate b. Case manager c. Milieu manager d. Provider of care

ANS: B Nurses on psychiatric units routinely coordinate patient services, serving as case managers as described in this scenario. The role of advocate would require the nurse to speak out on the patient's behalf. The role of milieu manager refers to maintaining a therapeutic environment. Provider of care refers to giving direct care to the patient.

A nurse inspects an inpatient psychiatric unit and finds that exits are free of obstructions, no one is smoking, and the janitor's closet is locked. These observations relate to: a. coordinating care of patients. b. management of milieu safety. c. management of the interpersonal climate. d. use of therapeutic intervention strategies.

ANS: B Nursing staff are responsible for all aspects of milieu management. The observations mentioned in this question directly relate to the safety of the unit. The other options, although part of the nurse's concerns, are unrelated to the observations cited.

26. A patient diagnosed with schizophrenia has taken a conventional antipsychotic medication for a year. Hallucinations are less intrusive, but the patient continues to have apathy, poverty of thought, and social isolation. The nurse would expect a change to which medication? a. Haloperidol (Haldol) b. Olanzapine (Zyprexa) c. Chlorpromazine (Thorazine) d. Diphenhydramine (Benadryl)

ANS: B Olanzapine is a second-generation atypical antipsychotic that targets both positive and negative symptoms of schizophrenia. Haloperidol and chlorpromazine are conventional antipsychotics that target only positive symptoms. Diphenhydramine is an antihistamine.

10. A patient diagnosed with schizophrenia demonstrates little spontaneous movement and has waxy flexibility. The patient's activities of daily living are severely compromised. An appropriate outcome would be that the patient will: a. demonstrate increased interest in the environment by the end of week 1. b. perform self-care activities with coaching by the end of day 3. c. gradually take the initiative for self-care by the end of week 2. d. accept tube feeding without objection by day 2.

ANS: B Outcomes related to self-care deficit nursing diagnoses should deal with increasing ability to perform self-care tasks independently, such as feeding, bathing, dressing, and toileting. Performing the tasks with coaching by nursing staff denotes improvement over the complete inability to perform the tasks. The incorrect options are not directly related to self-care activities, difficult to measure, and unrelated to maintenance of nutrition.

Which assessment finding for a patient in the community deserves priority intervention by the psychiatric nurse? The patient: a. receives Social Security disability income plus a small check from a trust fund every month. b. was absent from two of six planned Alcoholics Anonymous meetings in the past 2 weeks. c. lives in an apartment with two patients who attend partial hospitalization programs. d. has a sibling who was recently diagnosed with a mental illness.

ANS: B Patients who use alcohol or illegal substances often become medication noncompliant. Medication noncompliance, along with the disorganizing influence of substances on cellular brain function, promotes relapse. The distracters do not suggest problems.

A new staff nurse completes an orientation to the psychiatric unit. This nurse will expect to ask an advanced practice nurse to perform which action for patients? a. Perform mental health assessment interviews. b. Prescribe psychotropic medication. c. Establish therapeutic relationships. d. Individualize nursing care plans.

ANS: B Prescriptive privileges are granted to master's-prepared nurse practitioners who have taken special courses on prescribing medication. The nurse prepared at the basic level performs mental health assessments, establishes relationships, and provides individualized care planning. Note that this question was also offered for Chapter 1.

Which activity is appropriate for a nurse engaged exclusively in community-based primary prevention? a. Medication follow-up b. Teaching parenting skills c. Substance abuse counseling d. Making a referral for family therapy

ANS: B Primary prevention activities are directed to healthy populations to provide information for developing skills that promote mental health. The distracters represent secondary or tertiary prevention activities.

Select the example of primary prevention. a. Assisting a person diagnosed with a serious mental illness to fill a pill-minder b. Helping school-age children identify and describe normal emotions c. Leading a psychoeducational group in a community care home d. Medicating an acutely ill patient who assaulted a staff person

ANS: B Primary preventions are directed at healthy populations with a goal of preventing health problems from occurring. Helping school-age children describe normal emotions people experience promotes coping, a skill that is needed throughout life. Assisting a person with serious and persistent mental illness to fill a pill-minder is an example of tertiary prevention. Medicating an acutely ill patient who assaulted a staff person is a secondary prevention. Leading a psychoeducational group in a community care home is an example of tertiary prevention.

Which entry in the medical record best meets the requirement for problem-oriented charting? a. "A: Pacing and muttering to self. P: Sensory perceptual alteration related to internal auditory stimulation. I: Given fluphenazine HCL (Prolixin) 2.5 mg po at 0900 and went to room to lie down. E: Calmer by 0930. Returned to lounge to watch TV." b. "S: States, 'I feel like I'm ready to blow up.' O: Pacing hall, mumbling to self. A: Auditory hallucinations. P: Offer haloperidol (Haldol) 2 mg po. I: Haloperidol (Haldol) 2 mg po given at 0900. E: Returned to lounge at 0930 and quietly watched TV." c. "Agitated behavior. D: Patient muttering to self as though answering an unseen person. A: Given haloperidol (Haldol) 2 mg po and went to room to lie down. E: Patient calmer. Returned to lounge to watch TV." d. "Pacing hall and muttering to self as though answering an unseen person. haloperidol (Haldol) 2 mg po administered at 0900 with calming effect in 30 minutes. Stated, 'I'm no longer bothered by the voices.'"

ANS: B Problem-oriented documentation uses the first letter of key words to organize data: S for subjective data, O for objective data, A for assessment, P for plan, I for intervention, and E for evaluation. The distracters offer examples of PIE charting, focus documentation, and narrative documentation.

"QSEN" refers to: a. Qualitative Standardized Excellence in Nursing b. Quality and Safety Education for Nurses c. Quantitative Effectiveness in Nursing d. Quick Standards Essential for Nurses

ANS: B QSEN represents national initiatives centered on patient safety and quality. The primary goal of QSEN is to prepare future nurses with the knowledge, skills, and attitudes to increase the quality, care, and safety in the health care setting in which they work.

1. A person has had difficulty keeping a job because of arguing with co-workers and accusing them of conspiracy. Today the person shouts, "They're all plotting to destroy me. Isn't that true?" Select the nurse's most therapeutic response. a. "Everyone here is trying to help you. No one wants to harm you." b. "Feeling that people want to destroy you must be very frightening." c. "That is not true. People here are trying to help you if you will let them." d. "Staff members are health care professionals who are qualified to help you."

ANS: B Resist focusing on content; instead, focus on the feelings the patient is expressing. This strategy prevents arguing about the reality of delusional beliefs. Such arguments increase patient anxiety and the tenacity with which the patient holds to the delusion. The other options focus on content and provide opportunity for argument.

A nurse wants to enhance growth of a patient by showing positive regard. The nurse's action most likely to achieve this goal is: a. making rounds daily. b. staying with a tearful patient. c. administering medication as prescribed. d. examining personal feelings about a patient.

ANS: B Staying with a crying patient offers support and shows positive regard. Administering daily medication and making rounds are tasks that could be part of an assignment and do not necessarily reflect positive regard. Examining feelings regarding a patient addresses the nurse's ability to be therapeutic.

A patient is fearful of riding on elevators. The therapist first rides an escalator with the patient. The therapist and patient then stand in an elevator with the door open for five minutes and later with the elevator door closed for five minutes. Which technique has the therapist used? a. Classic psychoanalytic therapy b. Systematic desensitization c. Rational emotive therapy d. Biofeedback

ANS: B Systematic desensitization is a form of behavior modification therapy that involves the development of behavior tasks customized to the patient's specific fears. These tasks are presented to the patient while using learned relaxation techniques. The patient is incrementally exposed to the fear.

19. A patient took trifluoperazine 30 mg po daily for 3 years. The clinic nurse notes that the patient grimaces and constantly smacks both lips. The patient's neck and shoulders twist in a slow, snakelike motion. Which problem would the nurse suspect? a. Agranulocytosis b. Tardive dyskinesia c. Tourette's syndrome d. Anticholinergic effects

ANS: B Tardive dyskinesia is a neuroleptic-induced condition involving the face, trunk, and limbs. Involuntary movements, such as tongue thrusting; licking; blowing; irregular movements of the arms, neck, and shoulders; rocking; hip jerks; and pelvic thrusts, are seen. These symptoms are frequently not reversible even when the drug is discontinued. The scenario does not present evidence consistent with the other disorders mentioned. Agranulocytosis is a blood disorder. Tourette's syndrome is a condition in which tics are present. Anticholinergic effects include dry mouth, blurred vision, flushing, constipation, and dry eyes.

As a patient diagnosed with a mental illness is being discharged from a facility, a nurse invites the patient to the annual staff picnic. What is the best analysis of this scenario? a. The invitation facilitates dependency on the nurse. b. The nurse's action blurs the boundaries of the therapeutic relationship. c. The invitation is therapeutic for the patient's diversional activity deficit. d. The nurse's action assists the patient's integration into community living.

ANS: B The invitation creates a social relationship rather than a therapeutic relationship.

Which component of treatment of mental illness is specifically recognized by Quality and Safety Education for Nurses (QSEN)? a. All genomes are unique. b. Care is centered on the patient. c. Healthy development is vital to mental health. d. Recovery occurs on a continuum from illness to health.

ANS: B The key areas of care promoted by QSEN are patient-centered care, teamwork and collaboration, evidence-based practice, quality improvement, safety, and informatics.

A student nurse says, "I don't need to interact with my patients. I learn what I need to know by observation." An instructor can best interpret the nursing implications of Sullivan's theory to this student by responding: a. "Interactions are required in order to help you develop therapeutic communication skills." b. "Nurses cannot be isolated. We must interact to provide patients with opportunities to practice interpersonal skills." c. "Observing patient interactions will help you formulate priority nursing diagnoses and appropriate interventions." d. "It is important to pay attention to patients' behavioral changes, because these signify adjustments in personality."

ANS: B The nurse's role includes educating patients and assisting them in developing effective interpersonal relationships. Mutuality, respect for the patient, unconditional acceptance, and empathy are cornerstones of Sullivan's theory. The nurse who does not interact with the patient cannot demonstrate these cornerstones. Observations provide only objective data. Priority nursing diagnoses usually cannot be accurately established without subjective data from the patient. The other distracters relate to Maslow and behavioral theory. This item relates to an audience response question.

A nurse influenced by Peplau's interpersonal theory works with an anxious, withdrawn patient. Interventions should focus on: a. rewarding desired behaviors. b. use of assertive communication. c. changing the patient's self-concept. d. administering medications to relieve anxiety.

ANS: B The nurse-patient relationship is structured to provide a model for adaptive interpersonal relationships that can be generalized to others. Helping the patient learn to use assertive communication will improve the patient's interpersonal relationships. The distracters apply to theories of cognitive, behavioral, and biological therapy.

The parent of a child diagnosed with schizophrenia tearfully asks the nurse, "What could I have done differently to prevent this illness?" Select the nurse's best response. a. "Although schizophrenia results from impaired family relationships, try not to feel guilty. No one can predict how a child will respond to parental guidance." b. "Schizophrenia is a biological illness resulting from changes in how the brain and nervous system function. You are not to blame for your child's illness." c. "There is still hope. Changing your parenting style can help your child learn to cope effectively with the environment." d. "Most mental illnesses result from genetic inheritance. Your genes are more at fault than your parenting."

ANS: B The parent's comment suggests feelings of guilt or inadequacy. The nurse's response should address these feelings as well as provide information. Patients and families need reassurance that the major mental disorders are biological in origin and are not the "fault" of parents. One distracter places the burden of having faulty genes on the shoulders of the parents. The other distracters are neither wholly accurate nor reassuring.

35. A client says, "Facebook has a new tracking capacity. If I use the Internet, Homeland Security will detain me as a terrorist." Select the nurse's best initial action. a. Tell the client, "Facebook is a safe website. You don't need to worry about Homeland Security." b. Tell the client, "You are in a safe place where you will be helped." c. Administer a prn dose of an antipsychotic medication. d. Tell the client, "You don't need to worry about that."

ANS: B The patient is experiencing paranoia and delusional thinking, which leads to fear. Explaining that the patient is in a safe place will help relieve the fear. It is not therapeutic to disagree or give advice. Medication will not relieve the immediate concern.

3. A patient diagnosed with schizophrenia says, "My co-workers are out to get me. I also saw two doctors plotting to kill me." How does this patient perceive the environment? a. Disorganized b. Dangerous c. Supportive d. Bizarre

ANS: B The patient sees the world as hostile and dangerous. This assessment is important because the nurse can be more effective by using empathy to respond to the patient. Data are not present to support any of the other options.

A nurse wants to assess an adult patient's recent memory. Which question would best yield the desired information? a. "Where did you go to elementary school?" b. "What did you have for breakfast this morning?" c. "Can you name the current president of the United States?" d. "A few minutes ago, I told you my name. Can you remember it?"

ANS: B The patient's recall of a meal provides evidence of recent memory. Two incorrect responses are useful to assess immediate and remote memory. The other distracter assesses the patient's fund of knowledge.

When a nursing student expresses concerns about how mental health nurses "lose all their nursing skills," the best response by the mental health nurse is: a. "Psychiatric nurses practice in safer environments than other specialties. Nurse-to-patient ratios must be better because of the nature of the patients' problems." b. "Psychiatric nurses use complex communication skills as well as critical thinking to solve multidimensional problems. I am challenged by those situations." c. "That's a misconception. Psychiatric nurses frequently use high technology monitoring equipment and manage complex intravenous therapies." d. "Psychiatric nurses do not have to deal with as much pain and suffering as medical-surgical nurses do. That appeals to me."

ANS: B The practice of psychiatric nursing requires a different set of skills than medical-surgical nursing, though there is substantial overlap. Psychiatric nurses must be able to help patients with medical as well as mental health problems, reflecting the holistic perspective these nurses must have. Nurse-patient ratios and workloads in psychiatric settings have increased, just like other specialties. Psychiatric nursing involves clinical practice, not just documentation. Psychosocial pain and suffering are as real as physical pain and suffering.

A patient had psychotherapy weekly for 5 months. The therapist used free association, dream analysis, and facilitated transference to help the patient understand conflicts and foster change. Select the term that applies to this method. a. Rational-emotive behavior therapy b. Psychodynamic psychotherapy c. Cognitive-behavioral therapy d. Operant conditioning

ANS: B The techniques are aspects of psychodynamic psychotherapy. The distracters use other techniques.

After formulating the nursing diagnoses for a new patient, what is a nurse's next action? a. Designing interventions to include in the plan of care b. Determining the goals and outcome criteria c. Implementing the nursing plan of care d. Completing the spiritual assessment

ANS: B The third step of the nursing process is planning and outcome identification. Outcomes cannot be determined until the nursing assessment is complete and nursing diagnoses have been formulated.

A parent says, "My 2-year-old child refuses toilet training and shouts 'No!' when given directions. What do you think is wrong?" Select the nurse's best reply. a. "Your child needs firmer control. It is important to set limits now." b. "This is normal for your child's age. The child is striving for independence." c. "There may be developmental problems. Most children are toilet trained by age 2." d. "Some undesirable attitudes are developing. A child psychologist can help you develop a plan."

ANS: B This behavior is typical of a child around the age of 2 years, whose developmental task is to develop autonomy. The distracters indicate the child's behavior is abnormal.

During the first interview with a parent whose child died in a car accident, the nurse feels empathic and reaches out to take the patient's hand. Select the correct analysis of the nurse's behavior. a. It shows empathy and compassion. It will encourage the patient to continue to express feelings. b. The gesture is premature. The patient's cultural and individual interpretation of touch is unknown. c. The patient will perceive the gesture as intrusive and overstepping boundaries. d. The action is inappropriate. Psychiatric patients should not be touched.

ANS: B Touch has various cultural and individual interpretations. Nurses should refrain from using touch until an assessment can be made regarding the way in which the patient will perceive touch. The other options present prematurely drawn conclusions.

A patient says to the nurse, "My father has been dead for over 10 years, but talking to you is almost as comforting as the talks he and I had when I was a child." Which term applies to the patient's comment? a. Superego b. Transference c. Reality testing d. Counter-transference

ANS: B Transference refers to feelings a patient has toward the health care workers that were originally held toward significant others in his or her life. Counter-transference refers to unconscious feelings that the health care worker has toward the patient. The superego represents the moral component of personality; it seeks perfection.

11. A nurse observes a catatonic patient standing immobile, facing the wall with one arm extended in a salute. The patient remains immobile in this position for 15 minutes, moving only when the nurse gently lowers the arm. What is the name of this phenomenon? a. Echolalia b. Waxy flexibility c. Depersonalization d. Thought withdrawal

ANS: B Waxy flexibility is the ability to hold distorted postures for extended periods of time, as though the patient were molded in wax. Echolalia is a speech pattern. Depersonalization refers to a feeling state. Thought withdrawal refers to an alteration in thinking.

Serious mental illness is characterized as: a. any mental illness of more than 2 weeks' duration. b. a major long-term mental illness marked by significant functional impairments. c. a mental illness accompanied by physical impairment and severe social problems. d. a major mental illness that cannot be treated to prevent deterioration of cognitive and social abilities.

ANS: B "Serious mental illness" has replaced the term "chronic mental illness." Global impairments in function are evident, particularly social. Physical impairments may be present. Serious mental illness can be treated, but remissions and exacerbations are part of the course of the illness.

A person diagnosed with a serious mental illness enters a shelter for the homeless. Which intervention should be the nurse's initial priority? a. Find supported employment. b. Develop a trusting relationship. c. Administer prescribed medication. d. Teach appropriate health care practices.

ANS: B Basic psychosocial needs do not change because a person is homeless. The first step in caring for health care needs is establishing rapport. Once a trusting relationship is established, the nurse pursues other interventions.

A homeless patient diagnosed with a serious mental illness became suspicious and delusional. Depot antipsychotic medication began, and housing was obtained in a local shelter. One month later, which statement by the patient indicates significant improvement? a. "They will not let me drink. They have many rules in the shelter." b. "I feel comfortable here. Nobody bothers me." c. "Those shots make my arm very sore." d. "Those people watch me a lot."

ANS: B Evaluation of a patient's progress is made based on patient satisfaction with the new health status and the health care team's estimation of improvement. For a formerly delusional patient to admit to feeling comfortable and free of being "bothered" by others denotes improvement in the patient's condition. The other options suggest that the patient is in danger of relapse.

After 5 years in a state hospital, an adult diagnosed with schizophrenia was discharged to the community. This patient now requires persistent direction to accomplish activities of daily living and expects others to provide meals and do laundry. The nurse assesses this behavior as the probable result of: a. side effects of antipsychotic medications. b. dependency caused by institutionalization. c. cognitive deterioration from schizophrenia. d. stress associated with acclimation to the community.

ANS: B Institutions tend to impede independent functioning; for example, daily activities are planned and directed by staff; others provide meals and only at set times. Over time, patients become dependent on the institution to meet their needs and adapt to being cared for rather than caring for themselves. When these patients return to the community, many continue to demonstrate passive behaviors despite efforts to promote. Cognitive dysfunction and antipsychotic side effects can make planning and carrying out activities more difficult, but the question is more suggestive of adjustment to institutional care and difficulty readjusting to independence instead.

Many persons brought before a criminal court have mental illness, have committed minor offenses, and are off medications. The judge consults the nurse at the local community mental health center for guidance about how to respond when handling such cases. Which advice from the nurse would be most appropriate? a. "Sometimes a little time in jail makes a person rethink what they've been doing and puts them back on the right track." b. "Sentencing such persons to participate in treatment instead of incarcerating them has been shown to reduce repeat offenses." c. "Arresting these people helps them in the long run. Sometimes we cannot hospitalize them, but in jail they will get their medication." d. "Research suggests that special mental health courts do not make much difference so far, but outpatient commitment does seem to help."

ANS: B Research supports the use of special mental health courts that can sentence mentally ill persons to treatment instead of jail. Jail exposes vulnerable mentally ill persons to criminals, victimization, and high levels of stimulation and stress. Incarceration can also interrupt eligibility for benefits or lead to the loss of housing and often provides lower-quality mental health treatment in other settings. Recidivism rates for both mentally ill and non-mentally ill offenders are relatively high, so it does not appear that incarceration necessarily leads people to behave more appropriately. In addition, a criminal record can leave them more desperate and with fewer options after release. Research indicates that outpatient commitment is less effective at improving the mental health of mentally ill persons than was expected.

For patients diagnosed with serious mental illness, what is the major advantage of case management? a. The case manager can modify traditional psychotherapy. b. With one coordinator of services, resources can be more efficiently used. c. The case manager can focus on social skills training and esteem building. d. Case managers bring groups of patients together to discuss common problems.

ANS: B The case manager coordinates the care and multiple referrals that so often confuse the seriously mentally ill patient and the patient's family. Case management promotes efficient use of services. The other options are lesser advantages or are irrelevant.

The sibling of a patient who was diagnosed with a serious mental illness asks why a case manager has been assigned. The nurse's reply should cite the major advantage of the use of case management as: a. "The case manager can modify traditional psychotherapy for homeless patients so that it is more flexible." b. "Case managers coordinate services and help with accessing them, making sure the patient's needs are met." c. "The case manager can focus on social skills training and esteem building in the real world where the patient lives." d. "Having a case manager has been shown to reduce hospitalizations, which prevents disruption and saves money."

ANS: B The case manager helps the patient gain entrance into the system of care, can coordinate multiple referrals that so often confuse the seriously mentally ill person and his family, and can help overcome obstacles to access and treatment participation. Case managers do not usually possess the credentials needed to provide psychotherapy or function as therapists. Case management promotes efficient use of services in general, but only Assertive Community Treatment (ACT) programming has been shown to reduce hospitalization (which the sibling might see as a disadvantage). Case managers operate in the community, but this is not the primary advantage of their services.

A patient diagnosed with a serious mental illness died suddenly at age 52. The patient lived in the community for 5 years without relapse and held supported employment the past 6 months. The distressed family asks, "How could this happen?" Which response by the nurse accurately reflects research and addresses the family's question? a. "A certain number of people die young from undetected diseases, and it's just one of those sad things that sometimes happen." b. "Mentally ill people tend to die much younger than others, perhaps because they do not take as good care of their health, smoke more, or are overweight." c. "We will have to wait for the autopsy to know what happened. There were some medical problems, but we were not expecting death." d. "We are all surprised. The patient had been doing so well and saw the nurse every other week."

ANS: B The family is in distress. Because they do not understand his death, they are less able to accept it and seek specific information to help them understand what happened. Persons with serious mental illness die an average of 25 years prematurely. Contributing factors include failing to provide for their own health needs (e.g. forgetting to take medicine), inability to access or pay for care, higher rates of smoking, poor diet, criminal victimization, and stigma. The most accurate answer indicates that seriously mentally ill people are at much higher risk of premature death for a variety of reasons. Staff would not have been surprised that the patient died prematurely, and they would not attribute his death to random, undetected medical problems. Although the cause of death will not be reliably established until the autopsy, this response fails to address the family's need for information.

A patient living independently had command hallucinations to shout warnings to neighbors. After a short hospitalization, the patient was prohibited from returning to the apartment. The landlord said, "You cause too much trouble." What problem is the patient experiencing? a. Grief b. Stigma c. Homelessness d. Nonadherence

ANS: B The inability to obtain shelter because of negative attitudes about mental illness is an example of stigma. Stigma is defined as damage to reputation, shame, and ridicule society places on mental illness. Data are not present to identify grief as a patient problem. Data do not suggest that the patient is actually homeless. See relationship to audience response question.

1. Which descriptors exemplify consistency regarding nurse-patient relationships? Select all that apply. a. Encouraging a patient to share initial impressions of staff b. Having the same nurse care for a patient on a daily basis c. Providing a schedule of daily activities to a patient d. Setting a time for regular sessions with a patient e. Offering solutions to a patient's problems

ANS: B, C, D Consistency implies predictability. Having the same nurse see the patient daily and provide a daily schedule of patient activities and a set time for regular sessions will help a patient predict what will happen during each day and develop a greater degree of security and comfort. Encouraging a patient to share initial impressions of staff and giving advice are not related to consistency and would not be considered a therapeutic intervention.

1. A patient cries as the nurse explores the patient's feelings about the death of a close friend. The patient sobs, "I shouldn't be crying like this. It happened a long time ago." Which responses by the nurse facilitate communication?Select all that apply. a. "Why do you think you are so upset?" b. "I can see that you feel sad about this situation." c. "The loss of a close friend is very painful for you." d. "Crying is a way of expressing the hurt you are experiencing." e. "Let's talk about something else because this subject is upsetting you."

ANS: B, C, D Reflecting ("I can see that you feel sad," "This is very painful for you") and giving information ("Crying is a way of expressing hurt") are therapeutic techniques. "Why" questions often imply criticism or seem intrusive or judgmental. They are difficult to answer. Changing the subject is a barrier to communication.

Select all that apply. A person diagnosed with serious mental illness has frequent relapses, usually precipitated by situational stressors such as running out of money or the absence of key staff at the mental health center. Which interventions would the nurse suggest to reduce the risk of stressors to cause relapse? a. Discourage potentially stressful activities such as groups or volunteer work. b. Develop written plans that will help the patient remember what to do in a crisis. c. Help the patient identify and anticipate events that are likely to be overwhelming. d. Encourage health-promoting activities such as exercise and getting adequate rest. e. Accompany the patient to a National Alliance on Mental Illness support group.

ANS: B, C, D, E Basic interventions for coping with crises involve anticipating crises where possible and then developing a plan with specific actions to take when faced with an overwhelming stressor. Written plans are helpful; it can be difficult for anyone, especially a person with cognitive or memory impairments, to develop or remember steps to take when under overwhelming stress. Health-promoting activities enhance a person's ability to cope with stress. As the name suggests, support groups help a person develop a support system, and they provide practical guidance from peers who learned from experience how to deal with issues the patient may be facing. Groups and volunteer work may involve a measure of stress but also provide benefits that help persons cope and should not be discouraged unless they are being done to excess.

A patient is very suspicious and states, "The FBI has me under surveillance." Which strategies should a nurse use when gathering initial assessment data about this patient? Select all that apply. a. Tell the patient that medication will help this type of thinking. b. Ask the patient, "Tell me about the problem as you see it." c. Seek information about when the problem began. d. Tell the patient, "Your ideas are not realistic." e. Reassure the patient, "You are safe here."

ANS: B, C, E During the assessment interview, the nurse should listen attentively and accept the patient's statements in a nonjudgmental way. Because the patient is suspicious and fearful, reassuring safety may be helpful, although trust is unlikely so early in the relationship. Saying that medication will help or telling the patient that the ideas are not realistic will undermine development of trust between the nurse and patient.

Which activities represent the art of nursing? Select all that apply. a. Administering medications on time to a group of patients b. Listening to a new widow grieve her husband's death c. Helping a patient obtain groceries from a food bank d. Teaching a patient about a new medication e. Holding the hand of a frightened patient

ANS: B, C, E Peplau described the science and art of professional nursing practice. The art component of nursing consists of the care, compassion, and advocacy nurses provide to enhance patient comfort and well-being. The science component of nursing involves the application of knowledge to understand a broad range of human problems and psychosocial phenomena, intervening to relieve patients' suffering and promote growth. See related audience response question.

A patient in the emergency department says, "Voices say someone is stalking me. They want to kill me because I developed a cure for cancer. I have a knife and will stab anyone who is a threat." Which aspects of the patient's mental health have the greatest and most immediate concern to the nurse? Select all that apply. a. Happiness b. Appraisal of reality c. Control over behavior d. Effectiveness in work e. Healthy self-concept

ANS: B, C, E The aspects of mental health of greatest concern are the patient's appraisal of and control over behavior. The appraisal of reality is inaccurate. There are auditory hallucinations, delusions of persecution, and delusions of grandeur. In addition, the patient's control over behavior is tenuous, as evidenced by the plan to stab anyone who seems threatening. A healthy self-concept is lacking, as evidenced by the delusion of grandeur. Data are not present to suggest that the other aspects of mental health (happiness and effectiveness in work) are of immediate concern.

1. A nurse assessed a patient who reluctantly participated in activities, answered questions with minimal responses, and rarely made eye contact. What information should be included when documenting the assessment? Select all that apply. a. The patient was uncooperative b. The patient's subjective responses c. Only data obtained from the patient's verbal responses d. A description of the patient's behavior during the interview e. Analysis of why the patient was unresponsive during the interview

ANS: B, D Both content and process of the interview should be documented. Providing only the patient's verbal responses would create a skewed picture of the patient. Writing that the patient was uncooperative is subjectively worded. An objective description of patient behavior would be preferable. Analysis of the reasons for the patient's behavior would be speculation, which is inappropriate.

What information is conveyed by nursing diagnoses? Select all that apply. a. Medical judgments about the disorder b. Unmet patient needs currently present c. Goals and outcomes for the plan of care d. Supporting data that validate the diagnoses e. Probable causes that will be targets for nursing interventions

ANS: B, D, E Nursing diagnoses focus on phenomena of concern to nurses rather than on medical diagnoses.

A novice nurse tells a mentor, "I want to convey to my patients that I am interested in them and that I want to listen to what they have to say." Which behaviors will be helpful in meeting the nurse's goal? Select all that apply. a. Sitting behind a desk, facing the patient b. Introducing self to a patient and identifying own role c. Maintaining control of discussions by asking direct questions d. Using facial expressions to convey interest and encouragement e. Assuming an open body posture and sometimes mirror imaging

ANS: B, D, E Trust is fostered when the nurse gives an introduction and identifies his or her role. Facial expressions that convey interest and encouragement support the nurse's verbal statements to that effect and strengthen the message. An open body posture conveys openness to listening to what the patient has to say. Mirror imaging enhances patient comfort. A desk would place a physical barrier between the nurse and patient. A face-to-face stance should be avoided when possible and a less intense 90- or 120-degree angle used to permit either party to look away without discomfort.

Which behavior shows that a nurse values autonomy? The nurse: a. suggests one-on-one supervision for a patient who has suicidal thoughts. b. informs a patient that the spouse will not be in during visiting hours. c. discusses options and helps the patient weigh the consequences. d. sets limits on a patient's romantic overtures toward the nurse.

ANS: C A high level of valuing is acting on one's belief. Autonomy is supported when the nurse helps a patient weigh alternatives and their consequences before the patient makes a decision. Autonomy or self-determination is not the issue in any of the other behaviors.

As a nurse escorts a patient being discharged after treatment for major depression, the patient gives the nurse a necklace with a heart pendant and says, "Thank you for helping mend my broken heart." Which is the nurse's best response? a. "Accepting gifts violates the policies and procedures of the facility." b. "I'm glad you feel so much better now. Thank you for the beautiful necklace." c. "I'm glad I could help you, but I can't accept the gift. My reward is seeing you with a renewed sense of hope." d. "Helping people is what nursing is all about. It's rewarding to me when patients recognize how hard we work."

ANS: C Accepting a gift creates a social rather than therapeutic relationship with the patient and blurs the boundaries of the relationship. A caring nurse will acknowledge the patient's gesture of appreciation, but the gift should not be accepted. See relationship to audience response question.

An adolescent asks a nurse conducting an assessment interview, "Why should I tell you anything? You'll just tell my parents whatever you find out." Which response by the nurse is appropriate? a. "That isn't true. What you tell us is private and held in strict confidence. Your parents have no right to know." b. "Yes, your parents may find out what you say, but it is important that they know about your problems." c. "What you say about feelings is private, but some things, like suicidal thinking, must be reported to the treatment team." d. "It sounds as though you are not really ready to work on your problems and make changes."

ANS: C Adolescents are very concerned with confidentiality. The patient has a right to know that most information will be held in confidence but that certain material must be reported or shared with the treatment team, such as threats of suicide, homicide, use of illegal drugs, or issues of abuse. The incorrect responses are not true, will not inspire the confidence of the patient, or are confrontational.

When a new bill introduced in Congress reduces funding for care of persons with mental illness, a group of nurses writes letters to their elected representatives in opposition to the legislation. Which role have the nurses fulfilled? a. Recovery b. Attending c. Advocacy d. Evidence-based practice

ANS: C An advocate defends or asserts another's cause, particularly when the other person lacks the ability to do that for self. Examples of individual advocacy include helping patients understand their rights or make decisions. On a community scale, advocacy includes political activity, public speaking, and publication in the interest of improving the human condition. Since funding is necessary to deliver quality programming for persons with mental illness, the letter- writing campaign advocates for that cause on behalf of patients who are unable to articulate their own needs.

A patient discloses several concerns and associated feelings. If the nurse wants to seek clarification, which comment would be appropriate? a. "What are the common elements here?" b. "Tell me again about your experiences." c. "Am I correct in understanding that . . ." d. "Tell me everything from the beginning."

ANS: C Asking, "Am I correct in understanding that..." permits clarification to ensure that both the nurse and patient share mutual understanding of the communication. Asking about common elements encourages comparison rather than clarification. The remaining responses are implied questions that suggest the nurse was not listening.

A suspicious, socially isolated patient lives alone, eats one meal a day at a local shelter, and spends the remaining daily food allowance on cigarettes. Select a community psychiatric nurse's best initial action. a. Explore ways to help the patient stop smoking. b. Report the situation to the manager of the shelter. c. Assess the patient's weight; determine foods and amounts eaten. d. Arrange hospitalization for the patient in order to formulate a new treatment plan.

ANS: C Assessment of biopsychosocial needs and general ability to live in the community is called for before any other action is taken. Both nutritional status and income adequacy are critical assessment parameters. A patient may be able to maintain adequate nutrition while eating only one meal a day. The rule is to assess before taking action. Hospitalization may not be necessary. Smoking cessation strategies can be pursued later.

A patient tells the nurse, "I don't think I'll ever get out of here." Select the nurse's most therapeutic response. a. "Don't talk that way. Of course you will leave here!" b. keep up the good work, and you certainly will." c. "You don't think you're making progress?" d. "Everyone feels that way sometimes."

ANS: C By asking if the patient does not believe that progress has been made, the nurse is reflecting by putting into words what the patient is hinting. By making communication more explicit, issues are easier to identify and resolve. The remaining options are non-therapeutic techniques. Telling the patient not to "talk that way" is disapproving. Saying that everyone feels that way at times minimizes feelings. Telling the patient that good work will always result in success is falsely reassuring.

Which comment best indicates that a patient perceived the nurse was caring? "My nurse: a. always asks me which type of juice I want to help me swallow my medication." b. explained my treatment plan to me and asked for my ideas about how to make it better." c. spends time listening to me talk about my problems. That helps me feel like I am not alone." d. told me that if I take all the medicines the doctor prescribes, then I will get discharged sooner."

ANS: C Caring evidences empathetic understanding as well as competency. It helps change pain and suffering into a shared experience, creating a human connection that alleviates feelings of isolation. The distracters give examples of statements that demonstrate advocacy or giving advice.

Which disorder is a culture-bound syndrome? a. Epilepsy b. Schizophrenia c. Running amok d. Major depression

ANS: C Culture-bound syndromes occur in specific sociocultural contexts and are easily recognized by people in those cultures. A syndrome recognized in parts of Southeast Asia is running amok, in which a person (usually a male) runs around engaging in furious, almost indiscriminate violent behavior.

What is the desirable outcome for the orientation stage of a nurse-patient relationship? The patient will demonstrate behaviors that indicate: a. self-responsibility and autonomy. b. a greater sense of independence. c. rapport and trust with the nurse. d. resolved transference.

ANS: C Development of rapport and trust is necessary before the relationship can progress to the working phase. Behaviors indicating a greater sense of independence, self-responsibility, and resolved transference occur in the working phase.

A category 5 tornado occurred in a community of 400 people resulting in destruction of many homes and businesses. In the 2 years after this disaster, 140 individuals were diagnosed with posttraumatic stress disorder (PTSD). Which term best applies to these newly diagnosed cases? a. Prevalence b. Co-morbidity c. Incidence d. Parity

ANS: C Incidence refers to the number of new cases of mental disorders in a healthy population within a given period of time. Prevalence describes the total number of cases, new and existing, in a given population during a specific period of time, regardless of when they became ill. Parity refers to equivalence, and legislation required insurers that provide mental health coverage to offer annual and lifetime benefits at the same level provided for medical/surgical coverage. Co-morbidity refers to having more than one mental disorder at a time.

A patient begins a new program to assist with building social skills. In which part of the plan of care should a nurse record the item, "Encourage patient to attend one psychoeducational group daily"? a. Assessment b. Analysis c. Implementation d. Evaluation

ANS: C Interventions are the nursing prescriptions to achieve the outcomes. Interventions should be specific.

The spouse of a patient diagnosed with schizophrenia says, "I don't understand how events from childhood have anything to do with this disabling illness." Which response by the nurse will best help the spouse understand the cause of this disorder? a. "Psychological stress is the basis of most mental disorders." b. "This illness results from developmental factors rather than stress." c. "Research shows that this condition more likely has a biological basis." d. "It must be frustrating for you that your spouse is sick so much of the time."

ANS: C Many of the most prevalent and disabling mental disorders have strong biological influences. Genetics are only one part of biological factors. Empathy does not address increasing the spouse's level of knowledge about the cause of the disorder. The other distracters are not established facts.

When a new patient is hospitalized, a nurse takes the patient on a tour, explains rules of the unit, and discusses the daily schedule. The nurse is engaged in: a. counseling. b. health teaching. c. milieu management. d. psychobiological intervention.

ANS: C Milieu management provides a therapeutic environment in which the patient can feel comfortable and safe while engaging in activities that meet the patient's physical and mental health needs. Counseling refers to activities designed to promote problem solving and enhanced coping and includes interviewing, crisis intervention, stress management, and conflict resolution. Health teaching involves identifying health education needs and giving information about these needs. Psychobiological interventions involve medication administration and monitoring response to medications.

21. The nurse assesses a patient diagnosed with schizophrenia. Which assessment finding would the nurse regard as a negative symptom of schizophrenia? a. Auditory hallucinations b. Delusions of grandeur c. Poor personal hygiene d. Psychomotor agitation

ANS: C Negative symptoms include apathy, anhedonia, poor social functioning, and poverty of thought. Poor personal hygiene is an example of poor social functioning. The distracters are positive symptoms of schizophrenia. See relationship to audience response question.

36. Which finding constitutes a negative symptom associated with schizophrenia? a. Hostility b. Bizarre behavior c. Poverty of thought d. Auditory hallucinations

ANS: C Negative symptoms include apathy, anhedonia, poor social functioning, and poverty of thought. Poor personal hygiene is an example of poor social functioning. The distracters are positive symptoms of schizophrenia. See relationship to audience response question.

A patient says, "I'm still on restriction, but I want to attend some off-unit activities. Would you ask the doctor to change my privileges?" What is the nurse's best response? a. "Why are you asking me when you're able to speak for yourself?" b. "I will be glad to address it when I see your doctor later today." c. "That's a good topic for you to discuss with your doctor." d. "Do you think you can't speak to a doctor?".

ANS: C Nurses should encourage patients to work at their optimal level of functioning. A nurse does not act for the patient unless it is necessary. Acting for a patient increases feelings of helplessness and dependency

Select the most appropriate label to complete this nursing diagnosis: ___________ related to feelings of shyness and poorly developed social skills as evidenced by watching television alone at home every evening. a. Deficient knowledge b. Ineffective coping c. Social isolation d. Powerlessness

ANS: C Nursing diagnoses are selected based on the etiological factors and assessment findings, or evidence. In this instance, the evidence shows social isolation that is caused by shyness and poorly developed social skills.

Operant conditioning is part of the treatment plan to encourage speech in a child who is nearly mute. Which technique applies? a. Encourage the child to observe others talking. b. Include the child in small group activities. c. Give the child a small treat for speaking. d. Teach the child relaxation techniques.

ANS: C Operant conditioning involves giving positive reinforcement for a desired behavior. Treats are rewards and reinforce speech through positive reinforcement.

A nurse consistently encourages patient to do his or her own activities of daily living (ADLs). If the patient is unable to complete an activity, the nurse helps until the patient is once again independent. This nurse's practice is most influenced by which theorist? a. Betty Neuman b. Patricia Benner c. Dorothea Orem d. Joyce Travelbee

ANS: C Orem emphasizes the role of the nurse in promoting self-care activities of the patient; this has relevance to the seriously and persistently mentally ill patient.

A patient diagnosed with schizophrenia had an exacerbation related to medication noncompliance and was hospitalized for 5 days. The patient's thoughts are now more organized, and discharge is planned. The patient's family says, "It's too soon for discharge. We will just go through all this again." The nurse should: a. ask the case manager to arrange a transfer to a long-term care facility. b. notify hospital security to handle the disturbance and escort the family off the unit. c. explain that the patient will continue to improve if the medication is taken regularly. d. contact the health care provider to meet with the family and explain the discharge rationale.

ANS: C Patients do not stay in a hospital until every symptom disappears. The nurse must assume responsibility to advocate for the patient's right to the least restrictive setting as soon as the symptoms are under control and for the right of citizens to control health care costs. The health care provider will use the same rationale. Shifting blame will not change the discharge. Security is unnecessary. The nurse can handle this matter.

9. A patient diagnosed with schizophrenia exhibits little spontaneous movement and demonstrates waxy flexibility. Which patient needs are of priority importance? a. Self-esteem b. Psychosocial c. Physiological d. Self-actualization

ANS: C Physiological needs must be met to preserve life. A patient with waxy flexibility must be fed by hand or tube, toileted, given range-of-motion exercises, and so forth to preserve physiological integrity. Higher level needs are of lesser concern.

A psychotherapist works with an anxious, dependent patient. Which strategy is most consistent with psychoanalytic psychotherapy? a. Identifying the patient's strengths and assets b. Praising the patient for describing feelings of isolation c. Focusing on feelings developed by the patient toward the therapist d. Providing psychoeducation and emphasizing medication adherence

ANS: C Positive or negative feelings of the patient toward the therapist indicate transference. Transference is a psychoanalytic concept that can be used to explore previously unresolved conflicts. The distracters relate to biological therapy and supportive psychotherapy. Use of psychoeducational materials is a common "homework" assignment used in cognitive therapy.

16. A patient diagnosed with schizophrenia has taken fluphenazine (Prolixin) 5 mg po bid for 3 weeks. The nurse now observes a shuffling propulsive gait, a mask-like face, and drooling. Which term applies to these symptoms? a. Neuroleptic malignant syndrome b. Hepatocellular effects c. Pseudoparkinsonism d. Akathisia

ANS: C Pseudoparkinsonism induced by antipsychotic medication mimics the symptoms of Parkinson's disease. It frequently appears within the first month of treatment and is more common with first-generation antipsychotic drugs. Hepatocellular effects would produce abnormal liver test results. Neuroleptic malignant syndrome is characterized by autonomic instability. Akathisia produces motor restlessness.

A patient usually watches television all day, seldom going out in the community or socializing with others. The patient says, "I don't know what to do with my free time." Which member of the treatment team would be most helpful to this patient? a. Psychologist b. Social worker c. Recreational therapist d. Occupational therapist

ANS: C Recreational therapists help patients use leisure time to benefit their mental health. Occupational therapists assist with a broad range of skills, including those for employment. Psychologists conduct testing and provide other patient services. Social workers focus on the patient's support system.

Which issues should a nurse address during the first interview with a patient with a psychiatric disorder? a. Trust, congruence, attitudes, and boundaries b. Goals, resistance, unconscious motivations, and diversion c. Relationship parameters, the contract, confidentiality, and termination d. Transference, countertransference, intimacy, and developing resources

ANS: C Relationship parameters, the contract, confidentiality, and termination are issues that should be considered during the orientation phase of the relationship. The remaining options are issues that are dealt with later.

A newly admitted patient diagnosed with major depression has gained 20 pounds over a few months and has suicidal ideation. The patient has taken an antidepressant medication for 1 week without remission of symptoms. Select the priority nursing diagnosis. a. Imbalanced nutrition: more than body requirements b. Chronic low self-esteem c. Risk for suicide d. Hopelessness

ANS: C Risk for suicide is the priority diagnosis when the patient has both suicidal ideation and a plan to carry out the suicidal intent. Imbalanced nutrition, hopelessness, and chronic low self-esteem may be applicable nursing diagnoses, but these problems do not affect patient safety as urgently as would a suicide attempt.

Which principle should guide the nurse in determining the extent of silence to use during patient interview sessions? a. A nurse is responsible for breaking silences. b. Patients withdraw if silences are prolonged. c. Silence can provide meaningful moments for reflection. d. Silence helps patients know that what they said was understood.

ANS: C Silence can be helpful to both participants by giving each an opportunity to contemplate what has transpired, weigh alternatives, and formulate ideas. A nurse breaking silences is not a principle related to silences. It is inaccurate to say that patients withdraw during long silences or that silence helps patients know that they are understood. Feedback helps patients know they have been understood.

The case manager plans to discuss the treatment plan with a patient's family. Select the case manager's first action. a. Determine an appropriate location for the conference. b. Support the discussion with examples of the patient's behavior. c. Obtain the patient's permission for the exchange of information. d. Determine which family members should participate in the conference.

ANS: C The case manager must respect the patient's right to privacy, which extends to discussions with family. Talking to family members is part of the case manager's role. Actions identified in the distracters occur after the patient has given permission.

15. A newly admitted patient diagnosed with schizophrenia says, "The voices are bothering me. They yell and tell me I am bad. I have got to get away from them." Select the nurse's most helpful reply. a. "Do you hear the voices often?" b. "Do you have a plan for getting away from the voices?" c. "I'll stay with you. Focus on what we are talking about, not the voices. " d. "Forget the voices and ask some other patients to play cards with you."

ANS: C Staying with a distraught patient who is hearing voices serves several purposes: ongoing observation, the opportunity to provide reality orientation, a means of helping dismiss the voices, the opportunity of forestalling an action that would result in self-injury, and general support to reduce anxiety. Asking if the patient hears voices is not particularly relevant at this point. Asking if the patient plans to "get away from the voices" is relevant for assessment purposes but is less helpful than offering to stay with the patient while encouraging a focus on their discussion. Suggesting playing cards with other patients shifts responsibility for intervention from the nurse to the patient and other patients.

After several therapeutic encounters with a patient who recently attempted suicide, which occurrence should cause the nurse to consider the possibility of countertransference? a. The patient's reactions toward the nurse seem realistic and appropriate. b. The patient states, "Talking to you feels like talking to my parents." c. The nurse feels unusually happy when the patient's mood begins to lift. d. The nurse develops a trusting relationship with the patient.

ANS: C Strong positive or negative reactions toward a patient or over-identification with the patient indicate possible countertransference. Nurses must carefully monitor their own feelings and reactions to detect countertransference and then seek supervision. Realistic and appropriate reactions from a patient toward a nurse are desirable. One incorrect response suggests transference. A trusting relationship with the patient is desirable. See relationship to audience response question.

Termination of a therapeutic nurse-patient relationship has been successful when the nurse: a. avoids upsetting the patient by shifting focus to other patients before the discharge. b. gives the patient a personal telephone number and permission to call after discharge. c. discusses with the patient changes that happened during the relationship and evaluates outcomes. d. offers to meet the patient for coffee and conversation three times a week after discharge.

ANS: C Summarizing and evaluating progress help validate the experience for the patient and the nurse and facilitate closure. Termination must be discussed; avoiding discussion by spending little time with the patient promotes feelings of abandonment. Successful termination requires that the relationship be brought to closure without the possibility of dependency-producing ongoing contact.

A college student received an invitation to attend the wedding of a close friend who lives across the country. The student is afraid of flying. Which type of therapy would be most helpful for this patient? a. Psychoanalysis b. Milieu therapy c. Systematic desensitization d. Short-term dynamic therapy

ANS: C Systematic desensitization is a type of therapy aimed at extinguishing a specific behavior, such as the fear of flying. Psychoanalysis and short-term dynamic therapy seek to uncover conflicts. Milieu therapy involves environmental factors.

A patient was hospitalized for 24 hours after a reaction to a psychotropic medication. While planning discharge, the case manager learned that the patient received a notice of eviction immediately prior to admission. Select the case manager's most appropriate action. A. Postpone the patient's discharge from the hospital. b. Contact the landlord who evicted the patient to further discuss the situation. c. Arrange a temporary place for the patient to stay until new housing can be arranged. d. Determine whether the adverse medication reaction was genuine because the patient had nowhere to live.

ANS: C The case manager should intervene by arranging temporary shelter for the patient until an apartment can be found. This activity is part of the coordination and delivery of services that falls under the case manager role. None of the other options is a viable alternative.

The unit secretary receives a phone call from the health insurer for a hospitalized patient. The caller seeks information about the patient's projected length of stay. How should the nurse instruct the unit secretary to handle the request? a. Obtain the information from the patient's medical record and relay it to the caller. b. Inform the caller that all information about patients is confidential. c. Refer the request for information to the patient's case manager. d. Refer the request to the health care provider.

ANS: C The case manager usually confers with insurers and provides the treatment team with information about available resources. The unit secretary should be mindful of patient confidentiality and should neither confirm that the patient is an inpatient nor disclose other information.

The parent of a 4-year-old rewards and praises the child for helping a younger sibling, being polite, and using good manners. The nurse supports this use of praise related to these behaviors. These qualities are likely to be internalized and become part of which system of the personality? a. Id b. Ego c. Superego d. Preconscious

ANS: C The superego contains the "thou shalts," or moral standards internalized from interactions with significant others. Praise fosters internalization of desirable behaviors. The id is the center of basic instinctual drives, and the ego is the mediator. The ego is the problem-solving and reality-testing portion of the personality that negotiates solutions with the outside world. The preconscious is a level of awareness from which material can be retrieved easily with conscious effort. This item relates to an audience response question.

An adult says, "I never know the answers," and "My opinion doesn't count." Which psychosocial crisis was unsuccessfully resolved for this adult? a. Initiative versus guilt b. Trust versus mistrust c. Autonomy versus shame and doubt d. Generativity versus self-absorption

ANS: C These statements show severe self-doubt, indicating that the crisis of gaining control over the environment was not met successfully. Unsuccessful resolution of the crisis of initiative versus guilt results in feelings of guilt. Unsuccessful resolution of the crisis of trust versus mistrust results in poor interpersonal relationships and suspicion of others. Unsuccessful resolution of the crisis of generativity versus self-absorption results in self-absorption that limits the ability to grow as a person.

Documentation in a patient's chart shows, "Throughout a 5-minute interaction, patient fidgeted and tapped left foot, periodically covered face with hands, and looked under chair while stating, 'I enjoy spending time with you.'" Which analysis is most accurate? a. The patient is giving positive feedback about the nurse's communication techniques. b. The nurse is viewing the patient's behavior through a cultural filter. c. The patient's verbal and nonverbal messages are incongruent. d. The patient is demonstrating psychotic behaviors.

ANS: C When a verbal message is not reinforced with nonverbal behavior, the message is confusing and incongruent. Some clinicians call it a "mixed message." It is inaccurate to say that the patient is giving positive feedback about the nurse's communication techniques. The concept of a cultural filter is not relevant to the situation because a cultural filter determines what we will pay attention to and what we will ignore. Data are insufficient to draw the conclusion that the patient is demonstrating psychotic behaviors.

Which service would be expected to provide resources 24 hours a day, 7 days a week if needed for persons with serious mental illness? a. Clubhouse model b. Cognitive Behavioral Therapy (CBT) c. Assertive Community Treatment (ACT) d. Cognitive Enhancement Therapy (CET)

ANS: C Assertive community treatment (ACT) involves consumers working with a multidisciplinary team that provides a comprehensive array of services. At least one member of the team is available 24 hours a day for crisis needs, and the emphasis is on treating the patient within his own environment.

The nurse wants to enroll a patient with poor social skills in a training program for patients diagnosed with schizophrenia. Which description accurately describes social skills training? a. Patients learn to improve their attention and concentration. b. Group leaders provide support without challenging patients to change. c. Complex interpersonal skills are taught by breaking them into simpler behaviors. d. Patients learn social skills by practicing them in a supported employment setting.

ANS: C In social skills training, complex interpersonal skills are taught by breaking them down into component behaviors that are covered in a stepwise fashion. Social skills training is not based in employment settings, although such skills can be addressed as part of supported employment services. The other distracters are less relevant to social skills training.

Which nursing diagnosis is likely to apply to an individual diagnosed with a serious mental illness who is homeless? a. Insomnia b. Substance abuse c. Chronic low self-esteem d. Impaired environmental interpretation syndrome

ANS: C Many individuals with serious mental illness do not live with their families and become homeless. Life on the street or in a shelter has a negative influence on the individual's self-esteem, making this nursing diagnosis one that should be considered. Substance abuse is not an approved NANDA-International diagnosis. Insomnia may be noted in some patients but is not a universal problem. Impaired environmental interpretation syndrome refers to persistent disorientation, which is not seen in a majority of the homeless.

An adult diagnosed with a serious mental illness says, "I do not need help with money management. I have excellent ideas about investments." This patient usually does not have money to buy groceries by the middle of the month. The nurse assesses the patient as demonstrating: a. rationalization. b. identification. c. anosognosia. d. projection.

ANS: C The patient scenario describes anosognosia, the inability to recognize one's deficits due to one's illness. The patient is not projecting an undesirable thought or emotion from himself onto others. He is not justifying his behavior via rationalization and is not identifying with another.

Which comments by a nurse demonstrate use of therapeutic communication techniques? Select all that apply. a. "Why do you think these events have happened to you?" b. "There are people with problems much worse than yours." c. "I'm glad you were able to tell me how you felt about your loss." d. "I noticed your hands trembling when you told me about your accident." e. "You look very nice today. I'm proud you took more time with your appearance."

ANS: C, D The correct responses demonstrate use of the therapeutic techniques making an observation and showing empathy. The incorrect responses demonstrate minimizing feelings, probing, and giving approval, which are non-therapeutic techniques.

Which statements most clearly reflect the stigma of mental illness? Select all that apply. a. "Many mental illnesses are hereditary." b. "Mental illness can be evidence of a brain disorder." c. "People claim mental illness so they can get disability checks." d. "Mental illness results from the breakdown of American families." e. "If people with mental illness went to church, their symptoms would vanish."

ANS: C, D, E Stigma is represented by judgmental remarks that discount the reality and validity of mental illness. Many mental illnesses are genetically transmitted. Neuroimaging can show changes associated with some mental illnesses.

Select all that apply. A person diagnosed with a serious mental illness (SMI) living in the community was punched, pushed to the ground, and robbed of $7 during the day on a public street. Which statements about violence and serious mental illness in general are accurate? a. Persons with SMI are more likely to be violent. b. SMI persons are more likely to commit crimes than to be the victims of crime. c. Impaired judgment and social skills can provoke hostile or assaultive behavior. d. Lower incomes force SMI persons to live in high-crime areas, increasing risk. e. SMI persons experience higher rates of sexual assault and victimization than others. f. Criminals may believe SMI persons are less likely to resist or testify against them.

ANS: C, D, E, F Mentally ill persons are more likely to be victims of crime than perpetrators of criminal acts. They are often victims of criminal behavior, including sexual crimes, at a higher rate than others. When a mentally ill person commits a crime, it is usually nonviolent. Mental illnesses interfere with employment and are associated with poverty, limiting SMI persons to living in inexpensive areas that also tend to be higher-crime areas. SMI persons may inadvertently provoke others because of poor judgment or socially inappropriate behavior, or they may be victimized because they are perceived as passive, less likely to resist, and less likely to be believed as witnesses. See related audience response question.

A nurse interacts with a newly hospitalized patient. Select the nurse's comment that applies the communication technique of "offering self." a. "I've also had traumatic life experiences. Maybe it would help if I told you about them." b. "Why do you think you had so much difficulty adjusting to this change in your life?" c. "I hope you will feel better after getting accustomed to how this unit operates." d. "I'd like to sit with you for a while to help you get comfortable talking to me."

ANS: D "Offering self" is a technique that should be used in the orientation phase of the nurse-patient relationship. Sitting with the patient, an example of "offering self," helps to build trust and convey that the nurse cares about the patient. Two incorrect responses are ineffective and non-therapeutic. The other incorrect response is therapeutic but is an example of "offering hope."

Which belief will best support a nurse's efforts to provide patient advocacy during a multidisciplinary patient care planning session? a. All mental illnesses are culturally determined. b. Schizophrenia and bipolar disorder are cross-cultural disorders. c. Symptoms of mental disorders are unchanged from culture to culture. d. Assessment findings in mental disorders reflect a person's cultural patterns.

ANS: D A nurse who understands that a patient's symptoms are influenced by culture will be able to advocate for the patient to a greater degree than a nurse who believes that culture is of little relevance. The distracters are untrue statements.

The patients below were evaluated in the emergency department. The psychiatric unit has one bed available. Which patient should be admitted? The patient: a. feeling anxiety and a sad mood after separation from a spouse of 10 years. b. who self-inflicted a superficial cut on the forearm after a family argument. c. experiencing dry mouth and tremor related to taking haloperidol (Haldol). d. who is a new parent and hears voices saying, "Smother your baby."

ANS: D Admission to the hospital would be justified by the risk of patient danger to self or others. The other patients have issues that can be handled with less restrictive alternatives than hospitalization.

The desired outcome for a patient experiencing insomnia is, "Patient will sleep for a minimum of 5 hours nightly within 7 days." At the end of 7 days, review of sleep data shows the patient sleeps an average of 4 hours nightly and takes a 2-hour afternoon nap. The nurse will document the outcome as: a. consistently demonstrated. b. often demonstrated. c. sometimes demonstrated. d. never demonstrated.

ANS: D Although the patient is sleeping 6 hours daily, the total is not one uninterrupted session at night. Therefore, the outcome must be evaluated as never demonstrated. See relationship to audience response question.

The nurse assigned to assertive community treatment (ACT) should explain the program's treatment goal as: a. assisting patients to maintain abstinence from alcohol and other substances of abuse. b. providing structure and a therapeutic milieu for mentally ill patients whose symptoms require stabilization. c. maintaining medications and stable psychiatric status for incarcerated inmates who have a history of mental illness. d. providing services for mentally ill individuals who require intensive treatment to continue to live in the community.

ANS: D An assertive community treatment (ACT) program provides intensive community services to persons with serious, persistent mental illness who live in the community but require aggressive services to prevent repeated hospitalizations.

7. A health care provider considers which antipsychotic medication to prescribe for a patient diagnosed with schizophrenia who has auditory hallucinations and poor social function. The patient is also overweight and hypertensive. Which drug should the nurse advocate? a. Clozapine (Clozaril) b. Ziprasidone (Geodon) c. Olanzapine (Zyprexa) d. Aripiprazole (Abilify)

ANS: D Aripiprazole is a third-generation atypical antipsychotic effective against both positive and negative symptoms of schizophrenia. It causes little or no weight gain and no increase in glucose, high- or low-density lipoprotein cholesterol, or triglycerides, making it a reasonable choice for a patient with obesity or heart disease. Clozapine may produce agranulocytosis, making it a poor choice as a first-line agent. Ziprasidone may prolong the QT interval, making it a poor choice for a patient with cardiac disease. Olanzapine fosters weight gain.

Which patient would be most appropriate to refer for assertive community treatment (ACT)? A patient diagnosed with: a. a phobic fear of crowded places. b. a single episode of major depression. c. a catastrophic reaction to a tornado in the community. d. schizophrenia and four hospitalizations in the past year.

ANS: D Assertive community treatment (ACT) provides intensive case management for persons with serious persistent mental illness who live in the community. Repeated hospitalization is a frequent reason for this intervention. The distracters identify mental health problems of a more episodic nature.

Which aspect of direct care is an experienced, inpatient psychiatric nurse most likely to provide for a patient? a. Hygiene assistance b. Diversional activities c. Assistance with job hunting d. Building assertiveness skills

ANS: D Assertiveness training relies on the counseling and psychoeducational skills of the nurse. Assistance with personal hygiene would usually be accomplished by a psychiatric technician or nursing assistant. Diversional activities are usually the province of recreational therapists. The patient would probably be assisted in job hunting by a social worker or vocational therapist.

A nurse listens to a group of recent retirees. One says, "I volunteer with Meals on Wheels, coach teen sports, and do church visitation." Another laughs and says, "I'm too busy taking care of myself to volunteer to help others." Which developmental task do these statements contrast? a. Trust and mistrust b. Intimacy and isolation c. Industry and inferiority d. Generativity and self-absorption

ANS: D Both retirees are in middle adulthood, when the developmental crisis to be resolved is generativity versus self-absorption. One exemplifies generativity; the other embodies self-absorption. This developmental crisis would show a contrast between relating to others in a trusting fashion and being suspicious and lacking trust. Failure to negotiate this developmental crisis would result in a sense of inferiority or difficulty learning and working as opposed to the ability to work competently. Behaviors that would be contrasted would be emotional isolation and the ability to love and commit oneself.

A nurse is part of a multidisciplinary team working with groups of depressed patients. Half the patients receive supportive interventions and antidepressant medication. The other half receives only medication. The team measures outcomes for each group. Which type of study is evident? a. Incidence b. Prevalence c. Co-morbidity d. Clinical epidemiology

ANS: D Clinical epidemiology is a broad field that addresses studies of the natural history (or what happens if there is no treatment and the problem is left to run its course) of an illness, studies of diagnostic screening tests, and observational and experimental studies of interventions used to treat people with the illness or symptoms. Prevalence refers to numbers of new cases. Co-morbidity refers to having more than one mental disorder at a time.Incidence refers to the number of new cases of mental disorders in a healthy population within a given period. See related audience response question.

Clinical pathways are used in managed care settings to: a. stabilize aggressive patients. b. identify obstacles to effective care. c. relieve nurses of planning responsibilities. d. streamline the care process and reduce costs.

ANS: D Clinical pathways provide guidelines for assessments, interventions, treatments, and outcomes as well as a designated timeline for accomplishment. Deviations from the timeline must be reported and investigated. Clinical pathways streamline the care process and save money. Care pathways do not identify obstacles or stabilize aggressive patients. Staff are responsible for the necessary interventions. Care pathways do not relieve nurses of the responsibility of planning; pathways may, however, make the task easier.

A patient expresses a desire to be cared for by others and often behaves in a helpless fashion. Which stage of psychosexual development is most relevant to the patient's needs? a. Latency b. Phallic c. Anal d. Oral

ANS: D Fixation at the oral stage sometimes produces dependent infantile behaviors in adults. Latency fixations often result in difficulty identifying with others and developing social skills, resulting in a sense of inadequacy and inferiority. Phallic fixations result in having difficulty with authority figures and poor sexual identity. Anal fixation sometimes results in retentiveness, rigidity, messiness, destructiveness, and cruelty. This item relates to an audience response question.

34. The nurse is developing a plan for psychoeducational sessions for several adults diagnosed with schizophrenia. Which goal is best for this group? Members will: a. gain insight into unconscious factors that contribute to their illness. b. explore situations that trigger hostility and anger. c. learn to manage delusional thinking. d. demonstrate improved social skills.

ANS: D Improved social skills help patients maintain relationships with others. These relationships are important to management of the disorder. Most patients with schizophrenia think concretely, so insight development is unlikely. Not all patients with schizophrenia experience delusions.

A person says, "I was the only survivor in a small plane crash. Three business associates died. I got depressed and saw a counselor twice a week for 4 weeks. We talked about my feelings related to being a survivor, and I'm better now." Which type of therapy was used? a. Milieu therapy b. Psychoanalysis c. Behavior modification d. Interpersonal psychotherapy

ANS: D Interpersonal psychotherapy returned the patient to his former level of functioning by helping him come to terms with the loss of friends and guilt over being a survivor. Milieu therapy refers to environmental therapy. Psychoanalysis would call for a long period of exploration of unconscious material. Behavior modification would focus on changing a behavior rather than helping the patient understand what is going on in his life.

31. A patient diagnosed with schizophrenia and auditory hallucinations anxiously tells the nurse, "The voice is telling me to do things." Select the nurse's priority assessment question. a. "How long has the voice been directing your behavior?" b. "Does what the voice tell you to do frighten you?" c. "Do you recognize the voice speaking to you?' d. "What is the voice telling you to do?"

ANS: D Learning what a command hallucination is telling the patient to do is important because the command often places the patient or others at risk for harm. Command hallucinations can be terrifying and may pose a psychiatric emergency. The incorrect questions are of lesser importance than identifying the command.

25. A patient diagnosed with schizophrenia says, "It's beat. Time to eat. No room for the cat." What type of verbalization is evident? a. Neologism b. Idea of reference c. Thought broadcasting d. Associative looseness

ANS: D Looseness of association refers to jumbled thoughts incoherently expressed to the listener. Neologisms are newly coined words. Ideas of reference are a type of delusion. Thought broadcasting is the belief that others can hear one's thoughts.

A Puerto Rican American patient uses dramatic body language when describing emotional discomfort. Which analysis most likely explains the patient's behavior? The patient: a. has a histrionic personality disorder. b. believes dramatic body language is sexually appealing. c. wishes to impress staff with the degree of emotional pain. d. belongs to a culture in which dramatic body language is the norm.

ANS: D Members of Hispanic American subcultures tend to use high affect and dramatic body language as they communicate. The other options are more remote possibilities.

Nursing behaviors associated with the implementation phase of nursing process are concerned with: a. participating in mutual identification of patient outcomes. b. gathering accurate and sufficient patient-centered data. c. comparing patient responses and expected outcomes. d. carrying out interventions and coordinating care.

ANS: D Nursing behaviors relating to implementation include using available resources, performing interventions, finding alternatives when necessary, and coordinating care with other team members.

Which individual is demonstrating the highest level of resilience? One who: a. is able to repress stressors. b. becomes depressed after the death of a spouse. c. lives in a shelter for two years after the home is destroyed by fire. d. takes a temporary job to maintain financial stability after loss of a permanent job.

ANS: D Resilience is closely associated with the process of adapting and helps people facing tragedies, loss, trauma, and severe stress. It is the ability and capacity for people to secure the resources they need to support their well-being. Repression and depression are unhealthy. Living in a shelter for two years shows a failure to move forward after a tragedy. See related audience response question.

28. A patient diagnosed with schizophrenia has been stable for a year; however, the family now reports the patient is tense, sleeps 3 to 4 hours per night, and has difficulty concentrating. The patient says, "My computer is sending out infected radiation beams." The nurse can correctly assess this information as an indication of: a. the need for psychoeducation. b. medication noncompliance. c. chronic deterioration. d. relapse.

ANS: D Signs of potential relapse include feeling tense, difficulty concentrating, trouble sleeping, increased withdrawal, and increased bizarre or magical thinking. Medication noncompliance may not be implicated. Relapse can occur even when the patient is taking medication regularly. Psychoeducation is more effective when the patient's symptoms are stable. Chronic deterioration is not the best explanation.

The desired outcome for a patient experiencing insomnia is, "Patient will sleep for a minimum of 5 hours nightly within 7 days." At the end of 7 days, review of sleep data shows the patient sleeps an average of 4 hours nightly and takes a 2-hour afternoon nap. What is the nurse's next action? a. Continue the current plan without changes. b. Remove this nursing diagnosis from the plan of care. c. Write a new nursing diagnosis that better reflects the problem. d. Examine interventions for possible revision of the target date.

ANS: D Sleeping a total of 5 hours at night remains a reasonable outcome. Extending the period for attaining the outcome may be appropriate. Examining interventions might result in planning an activity during the afternoon rather than permitting a nap. Continuing the current plan without changes is inappropriate. Removing this nursing diagnosis from the plan of care would be correct when the outcome was met and the problem resolved. Writing a new nursing diagnosis is inappropriate because no other nursing diagnosis relates to the problem.

Although ego defense mechanisms and security operations are mainly unconscious and designed to relieve anxiety, the major difference is that: a. defense mechanisms are intrapsychic and not observable. b. defense mechanisms cause arrested personal development. c. security operations are masterminded by the id and superego. d. security operations address interpersonal relationship activities.

ANS: D Sullivan's theory explains that security operations are interpersonal relationship activities designed to relieve anxiety. Because they are interpersonal, they are observable. Defense mechanisms are unconscious and automatic. Repression is entirely intrapsychic, but other mechanisms result in observable behaviors. Frequent, continued use of many defense mechanisms often results in reality distortion and interference with healthy adjustment and emotional development. Occasional use of defense mechanisms is normal and does not markedly interfere with development. Security operations are ego-centered. This item relates to an audience response question.

A citizen at a community health fair asks the nurse, "What is the most prevalent mental disorder in the United States?" Select the nurse's best response. a. Schizophrenia b. Bipolar disorder c. Dissociative fugue d. Alzheimer's disease

ANS: D The 12-month prevalence for Alzheimer's disease is 10% for persons older than 65 and 50% for persons older than 85. The prevalence of schizophrenia is 1.1% per year. The prevalence of bipolar disorder is 2.6%. Dissociative fugue is a rare disorder. See related audience response question.

The Diagnostic and Statistical Manual of Mental Disorders (DSM-5) classifies: a. deviant behaviors. b. present disability or distress. c. people with mental disorders. d. mental disorders people have.

ANS: D The DSM-5 classifies disorders people have rather than people themselves. The terminology of the tool reflects this distinction by referring to individuals with a disorder rather than as a "schizophrenic" or "alcoholic," for example. Deviant behavior is not generally considered a mental disorder. Present disability or distress is only one aspect of the diagnosis.

A health care provider prescribed depot injections every 3 weeks at the clinic for a patient with a history of medication noncompliance. For this plan to be successful, which factor will be of critical importance? a. The attitude of significant others toward the patient b. Nutrition services in the patient's neighborhood c. The level of trust between the patient and nurse d. The availability of transportation to the clinic

ANS: D The ability of the patient to get to the clinic is of paramount importance to the success of the plan. The depot medication relieves the patient of the necessity to take medication daily, but if he or she does not receive the injection at 3-week intervals, non-adherence will again be the issue. Attitude toward the patient, trusting relationships, and nutrition are important but not fundamental to this particular problem.

A Filipino American patient had a nursing diagnosis of situational low self-esteem related to poor social skills as evidenced by lack of eye contact. Interventions were used to raise the patient's self-esteem, but after 3 weeks, the patient's eye contact did not improve. What is the most accurate analysis of this scenario? a. The patient's eye contact should have been directly addressed by role-playing to increase comfort with eye contact. b. The nurse should not have independently embarked on assessment, diagnosis, and planning for this patient. c. The patient's poor eye contact is indicative of anger and hostility that were unaddressed. d. The nurse should have assessed the patient's culture before making this diagnosis and plan.

ANS: D The amount of eye contact a person engages in is often culturally determined. In some cultures, eye contact is considered insolent, whereas in others eye contact is expected. Asian Americans, including persons from the Philippines, often prefer not to engage in direct eye contact.

A school age child tells the school nurse, "Other kids call me mean names and will not sit with me at lunch. Nobody likes me." Select the nurse's most therapeutic response. a. "Just ignore them and they will leave you alone." b. "You should make friends with other children." c. "Call them names if they do that to you." d. "Tell me more about how you feel."

ANS: D The correct response uses exploring, a therapeutic technique. The distracters give advice, a non-therapeutic technique.

A 26-month-old displays negative behavior, refuses toilet training, and often says, "No!" Which psychosocial crisis is evident? a. Trust versus mistrust b. Initiative versus guilt c. Industry versus inferiority d. Autonomy versus shame and doubt

ANS: D The crisis of autonomy versus shame and doubt relates to the developmental task of gaining control of self and environment, as exemplified by toilet training. This psychosocial crisis occurs during the period of early childhood. Trust versus mistrust is the crisis of the infant. Initiative versus guilt is the crisis of the preschool and early-school-aged child. Industry versus inferiority is the crisis of the 6- to 12-year-old child.

A nurse documents: "Patient is mute despite repeated efforts to elicit speech. Makes no eye contact. Inattentive to staff. Gazes off to the side or looks upward rather than at speaker." Which nursing diagnosis should be considered? a. Defensive coping b. Decisional conflict c. Risk for other-directed violence d. Impaired verbal communication

ANS: D The defining characteristics are more related to the nursing diagnosis of impaired verbal communication than to the other nursing diagnoses.

A nurse supports a parent for praising a child behaving in a helpful way. When this child behaves with politeness and helpfulness in adulthood, which feeling will most likely result? a. Guilt b. Anxiety c. Humility d. Self-esteem

ANS: D The individual will be living up to the ego ideal, which will result in positive feelings about self. The other options are incorrect because each represents a negative feeling.

A patient's relationships are intense and unstable. The patient initially idealizes the significant other and then devalues him or her, resulting in frequent feelings of emptiness. This patient will benefit from interventions to develop which aspect of mental health? a. Effectiveness in work b. Communication skills c. Productive activities d. Fulfilling relationships

ANS: D The information given centers on relationships with others that are described as intense and unstable. The relationships of mentally healthy individuals are stable, satisfying, and socially integrated. Data are not present to describe work effectiveness, communication skills, or activities.

Select the best response for the nurse who receives a question from another health professional seeking to understand the difference between a Diagnostic and Statistical Manual of Mental Disorders (DSM-5) diagnosis and a nursing diagnosis. a. "There is no functional difference between the two. Both identify human disorders." b. "The DSM-5 diagnosis disregards culture, whereas the nursing diagnosis takes culture into account. c. The DSM-5 diagnosis describes causes of disorders whereas a nursing diagnosis does not explore etiology." d. "The DSM-5 diagnosis guides medical treatment, whereas the nursing diagnosis offers a framework for identifying interventions for issues a patient is experiencing."

ANS: D The medical diagnosis is concerned with the patient's disease state, causes, and cures, whereas the nursing diagnosis focuses on the patient's response to stress and possible caring interventions. Both tools consider culture. The DSM-5 is multiaxial. Nursing diagnoses also consider potential problems.

A nurse caring for a withdrawn, suspicious patient recognizes development of feelings of anger toward the patient. The nurse should: a. suppress the angry feelings. b. express the anger openly and directly with the patient. c. tell the nurse manager to assign the patient to another nurse. d. discuss the anger with a clinician during a supervisory session.

ANS: D The nurse is accountable for the relationship. Objectivity is threatened by strong positive or negative feelings toward a patient. Supervision is necessary to work through countertransference feelings.

A nurse prepares to assess a new patient who moved to the United States from Central America three years ago. After introductions, what is the nurse's next comment? a. "How did you get to the United States?" b. "Would you like for a family member to help you talk with me?" c. "An interpreter is available. Would you like for me to make a request for these services?" d. "Are you comfortable conversing in English, or would you prefer to have a translator present?"

ANS: D The nurse should determine whether a translator is needed by first assessing the patient for language barriers. Accuracy of the assessment depends on the ability to communicate in a language that is familiar to the patient. Family members are not always reliable translators. An interpreter may change the patient's responses; a translator is a better resource.

A patient states, "I'm not worth anything. I have negative thoughts about myself. I feel anxious and shaky all the time. Sometimes I feel so sad that I want to go to sleep and never wake up." Which nursing intervention should have the highest priority? a. Self-esteem-building activities b. Anxiety self-control measures c. Sleep enhancement activitie d. Suicide precautions

ANS: D The nurse would place a priority on monitoring and reinforcing suicide self-restraint because it relates directly and immediately to patient safety. Patient safety is always a priority concern. The nurse should monitor and reinforce all patient attempts to control anxiety, improve sleep patterns, and develop self-esteem, while giving priority attention to suicide self-restraint.

Select the best outcome for a patient with the nursing diagnosis: Impaired social interaction related to sociocultural dissonance as evidenced by stating, "Although I'd like to, I don't join in because I don't speak the language very well." Patient will: a. show improved use of language. b. demonstrate improved social skills. c. become more independent in decision making. d. select and participate in one group activity per day.

ANS: D The outcome describes social involvement on the part of the patient. Neither cooperation nor independence has been an issue. The patient has already expressed a desire to interact with others. Outcomes must be measurable. Two of the distracters are not measurable.

30. A patient diagnosed with schizophrenia anxiously says, "I can see the left side of my body merging with the wall, then my face appears and disappears in the mirror." While listening, the nurse should: a. sit close to the patient. b. place an arm protectively around the patient's shoulders. c. place a hand on the patient's arm and exert light pressure. d. maintain a normal social interaction distance from the patient.

ANS: D The patient is describing phenomena that indicate personal boundary difficulties and depersonalization. The nurse should maintain appropriate social distance and not touch the patient because the patient is anxious about the inability to maintain ego boundaries and merging or being swallowed by the environment. Physical closeness or touch could precipitate panic.

20. A nurse sits with a patient diagnosed with schizophrenia. The patient starts to laugh uncontrollably, although the nurse has not said anything funny. Select the nurse's best response. a. "Why are you laughing?" b. "Please share the joke with me." c. "I don't think I said anything funny." d. "You're laughing. Tell me what's happening."

ANS: D The patient is likely laughing in response to inner stimuli, such as hallucinations or fantasy. Focus on the hallucinatory clue (the patient's laughter) and then elicit the patient's observation. The incorrect options are less useful in eliciting a response: no joke may be involved, "why" questions are difficult to answer, and the patient is probably not focusing on what the nurse said in the first place.

23. A patient diagnosed with schizophrenia says, "Contagious bacteria are everywhere. When they get in your body, you will be locked up with other infected people." Which problem is evident? a. Poverty of content b. Concrete thinking c. Neologisms d. Paranoia

ANS: D The patient's unrealistic fear of harm indicates paranoia. Neologisms are invented words. Concrete thinking involves literal interpretation. Poverty of content refers to an inadequate fund of information.

Which comment best indicates a patient is self-actualized? a. "I have succeeded despite a world filled with evil." b. "I have a plan for my life. If I follow it, everything will be fine." c. "I'm successful because I work hard. No one has ever given me anything." d. "My favorite leisure is walking on the beach, hearing soft sounds of rolling waves."

ANS: D The self-actualized personality is associated with high productivity and enjoyment of life. Self-actualized persons experience pleasure in being alone and an ability to reflect on events.

Which statement made by a patient during an initial assessment interview should serve as the priority focus for the plan of care? a. "I can always trust my family." b. "It seems like I always have bad luck." c. "You never know who will turn against you." d. "I hear evil voices that tell me to do bad things."

ANS: D The statement regarding evil voices tells the nurse that the patient is experiencing auditory hallucinations and may create risks for violence. The other statements are vague and do not clearly identify the patient's chief symptom.

A patient says to the nurse, "I dreamed I was stoned. When I woke up, I felt emotionally drained, as though I hadn't rested well." Which response should the nurse use to clarify the patient's comment? a. "It sounds as though you were uncomfortable with the content of your dream." b. "I understand what you're saying. Bad dreams leave me feeling tired, too." c. "So you feel as though you did not get enough quality sleep last night?" d. "Can you give me an example of what you mean by 'stoned'?"

ANS: D The technique of clarification is therapeutic and helps the nurse examine the meaning of the patient's statement. Asking for a definition of "stoned" directly asks for clarification. Restating that the patient is uncomfortable with the dream's content is parroting, a non-therapeutic technique. The other responses fail to clarify the meaning of the patient's comment.

Which remark by a patient indicates passage from orientation to the working phase of a nurse-patient relationship? a. "I don't have any problems." b. "It is so difficult for me to talk about problems." c. "I don't know how it will help to talk to you about my problems." d. "I want to find a way to deal with my anger without becoming violent."

ANS: D Thinking about a more constructive approach to dealing with anger indicates a readiness to make a behavioral change. Behavioral change is associated with the working phase of the relationship. Denial is often seen in the orientation phase. It is common early in the relationship, before rapport and trust are firmly established, for a patient to express difficulty in talking about problems. Stating skepticism about the effectiveness of the nurse-patient relationship is more typically a reaction during the orientation phase.

The nurse should refer which of the following patients to a partial hospitalization program? A patient who: a. has a therapeutic lithium level and reports regularly for blood tests and clinic follow-up. b. needs psychoeducation for relaxation therapy related to agoraphobia and panic episodes. c. spent yesterday in a supervised crisis care center and continues to have active suicidal ideation. d. states, "I'm not sure I can avoid using alcohol when my spouse goes to work every morning."

ANS: D This patient could profit from the structure and supervision provided by spending the day at the partial hospitalization program. During the evening, at night, and on weekends, the spouse could assume responsibility for supervision. A suicidal patient needs inpatient hospitalization. The other patients can be served in the community or with individual visits.

A patient says, "People should be allowed to commit suicide without interference from others." A nurse replies, "You're wrong. Nothing is bad enough to justify death." What is the best analysis of this interchange? a. The patient is correct. b. The nurse is correct. c. Neither person is correct. d. Differing values are reflected in the two statements.

ANS: D Values guide beliefs and actions. The individuals stating their positions place different values on life and autonomy. Nurses must be aware of their own values and be sensitive to the values of others.

During an interview, a patient attempts to shift the focus from self to the nurse by asking personal questions. The nurse should respond by saying: a. "Why do you keep asking about me?" b. "Nurses direct the interviews with patients." c. "Do not ask questions about my personal life." d. "The time we spend together is to discuss your concerns."

ANS: D When a patient tries to focus on the nurse, the nurse should refocus the discussion back onto the patient. Telling the patient that interview time should be used to discuss patient concerns refocuses discussion in a neutral way. Telling patients not to ask about the nurse's personal life shows indignation. Saying that nurses prefer to direct the interview reflects superiority. "Why" questions are probing and non-therapeutic.

8. A patient diagnosed with schizophrenia tells the nurse, "I eat skiller. Tend to end. Easter. It blows away. Get it?" Select the nurse's best response. a. "Nothing you are saying is clear." b. "Your thoughts are very disconnected." c. "Try to organize your thoughts and then tell me again." d. "I am having difficulty understanding what you are saying."

ANS: D When a patient's speech is loosely associated, confused, and disorganized, pretending to understand is useless. The nurse should tell the patient that he or she is having difficulty understanding what the patient is saying. If a theme is discernible, ask the patient to talk about the theme. The incorrect options tend to place blame for the poor communication with the patient. The correct response places the difficulty with the nurse rather than being accusatory. See relationship to audience response question.

A nurse is talking with a patient, and 5 minutes remain in the session. The patient has been silent most of the session. Another patient comes to the door of the room, interrupts, and says to the nurse, "I really need to talk to you." The nurse should: a. invite the interrupting patient to join in the session with the current patient. b. say to the interrupting patient, "I am not available to talk with you at the present time." c. end the unproductive session with the current patient and spend time with the interrupting patient. d. tell the interrupting patient, "This session is 5 more minutes; then I will talk with you."

ANS: D When a specific duration for sessions has been set, the nurse must adhere to the schedule. Leaving the first patient would be equivalent to abandonment and would destroy any trust the patient had in the nurse. Adhering to the contract demonstrates that the nurse can be trusted and that the patient and the sessions are important. The incorrect responses preserve the nurse-patient relationship with the silent patient but may seem abrupt to the interrupting patient, abandon the silent patient, or fail to observe the contract with the silent patient.

14. Withdrawn patients diagnosed with schizophrenia: a. are usually violent toward caregivers. b. universally fear sexual involvement with therapists. c. exhibit a high degree of hostility as evidenced by rejecting behavior. d. avoid relationships because they become anxious with emotional closeness.

ANS: D When an individual is suspicious and distrustful and perceives the world and the people in it as potentially dangerous, withdrawal into an inner world can be a defense against uncomfortable levels of anxiety. When someone attempts to establish a relationship with such a patient, the patient's anxiety rises until trust is established. There is no evidence that withdrawn patients with schizophrenia universally fear sexual involvement with therapists. In most cases, it is untrue that withdrawn patients with schizophrenia are commonly violent or exhibit a high degree of hostility by demonstrating rejecting behavior.

At what point in an assessment interview would a nurse ask, "How does your faith help you in stressful situations?" During the assessment of: a. childhood growth and development b. substance use and abuse c. educational background d. coping strategies

ANS: D When discussing coping strategies, the nurse might ask what the patient does when upset, what usually relieves stress, and to whom the patient goes to talk about problems. The question regarding whether the patient's faith helps deal with stress fits well here. It would be out of place if introduced during exploration of the other topics.

A nurse assesses an older adult patient brought to the emergency department by a family member. The patient was wandering outside saying, "I can't find my way home." The patient is confused and unable to answer questions. Select the nurse's best action. a. Record the patient's answers to questions on the nursing assessment form. b. Ask an advanced practice nurse to perform the assessment interview. c. Call for a mental health advocate to maintain the patient's rights. d. Obtain important information from the family member.

ANS: D When the patient (primary source) is unable to provide information, secondary sources should be used, in this case, the family member. Later, more data may be obtained from other information sources familiar with the patient. An advanced practice nurse is not needed for this assessment; it is within the scope of practice of the staff nurse. Calling a mental health advocate is unnecessary. See relationship to audience response question.

A homeless individual diagnosed with serious mental illness, anosognosia, and a history of persistent treatment nonadherence is persuaded to come to the day program at a community mental health center. Which intervention should be the team's initial focus? a. Teach appropriate health maintenance and prevention practices. b. Educate the patient about the importance of treatment adherence. c. Help the patient obtain employment in a local sheltered workshop. d. Interact regularly and supportively without trying to change the patient.

ANS: D Given the history of treatment nonadherence and the difficulty achieving other goals until psychiatrically stable and adherent, getting the patient to accept and adhere to treatment is the fundamental goal to address. The intervention most likely to help meet that goal at this stage is developing a trusting relationship with the patient. Interacting regularly, supportively, and without demands is likely to build the necessary trust and relationships that will be the foundation for all other interventions later on. No data here suggest the patient is in crisis, so it is possible to proceed slowly and build this foundation of trust.

A patient diagnosed with schizophrenia has had multiple relapses. The patient usually responds quickly to antipsychotic medication but soon discontinues the medication. Discharge plans include follow-up at the mental health center, group home placement, and a psychosocial day program. Which strategy should apply as the patient transitions from hospital to community? a. Administer a second-generation antipsychotic to help negative symptoms. b. Use a quick-dissolving medication formulation to reduce "cheeking." c. Prescribe a long-acting intramuscular antipsychotic medication. d. Involve the patient in decisions about which medication is best.

ANS: D Persons with schizophrenia are at high risk for treatment nonadherence, so the strategy needs primarily to address that risk. Of the options here, involving the patient in the decision is best because it will build trust and help establish a therapeutic alliance with care providers, an essential foundation to adherence. Intramuscular depot medications can be helpful for promoting adherence if other alternatives have been unsuccessful, but IM medications are painful and may jeopardize the patient's acceptance. All of the other strategies also apply but are secondary to trust and bonding with providers.

The parent of a seriously mentally ill adult asks the nurse, "Why are you making a referral to a vocational rehabilitation program? My child won't ever be able to hold a job." Which is the nurse's best reply? a. "We make this referral to continue eligibility for federal funding." b. "Are you concerned that we're trying to make your child too independent?" c. "If you think the program would be detrimental, we can postpone it for a time." d. "Most patients are capable of employment at some level, competitive or supported."

ANS: D Studies have shown that most patients who complete vocational rehabilitation programs are capable of some level of employment. They also demonstrate significant improvement in assertiveness and work behaviors as well as decreased depression.

A family discusses the impact of a seriously mental ill member. Insurance partially covered treatment expenses, but the family spent much of their savings for care. The patient's sibling says, "My parents have no time for me." The parents are concerned that when they are older, there will be no one to care for the patient. Which response by the nurse would be most helpful? a. Acknowledge their concerns and consult with the treatment team about ways to bring the patient's symptoms under better control. b. Give them names of financial advisors that could help them save or borrow sufficient funds to leave a trust fund to care for their loved one. c. Refer them to crisis intervention services to learn ways to manage caregiver stress and provide titles of some helpful books for families. d. Discuss benefits of participating in National Alliance on Mental Illness (NAMI) programs and ways to help the patient become more independent.

ANS: D The family has raised a number of concerns, but the major issues appear to be the effects caregiving has had on the family and their concerns about the patient's future. The National Alliance on Mental Illness (NAMI) offers support, education, resources, and access to other families who have experience with the issues now facing this family. NAMI can help address caregiver burden and planning for the future needs of SMI persons. Improving the patient's symptom control and general functioning can help reduce caregiver burden but would likely be a slow process, whereas NAMI involvement could benefit them on a number of fronts, possibly in a shorter time period. The family will need more than financial planning; their issues go beyond financial. The family is distressed but not in crisis. Crisis intervention is not an appropriate resource for the longer-term issues and needs affecting this family.

An outpatient diagnosed with schizophrenia attends programming at a community mental health center. The patient says, "I threw away the pills because they keep me from hearing God." Which response by the nurse would most likely to benefit this patient? a. "You need your medicine. Your schizophrenia will get worse without it." b. "Do you want to be hospitalized again? You must take your medication." c. "I would like you to come to the medication education group every Thursday." d. "I noticed that when you take the medicine, you have been able to hold a job you wanted."

ANS: D The patient appears not to understand that he has an illness. He has stopped his medication because it interferes with a symptom that he finds desirable (auditory hallucinations—the voice of God). Connecting medication adherence to one of the patient's goals (the job) can serve to motivate the patient to take the medication and override concerns about losing the hallucinations. Exhorting a patient to take medication because it is needed to control his illness is unlikely to be successful; he does not believe he has an illness. Medication psychoeducation would be appropriate if the cause of nonadherence was a knowledge deficit.

A patient diagnosed with schizophrenia tells the community mental health nurse, "I threw away my pills because they interfere with God's voice." The nurse identifies the etiology of the patient's ineffective management of the medication regime as: a. inadequate discharge planning. b. poor therapeutic alliance with clinicians. c. dislike of antipsychotic medication side effects. d. impaired reasoning secondary to the schizophrenia.

ANS: D The patient's ineffective management of the medication regime is most closely related to impaired reasoning. The patient believes in being an exalted personage who hears God's voice, rather than an individual with a serious mental disorder who needs medication to control symptoms. Data do not suggest any of the other factors often related to medication nonadherence.

An outpatient diagnosed with schizophrenia tells the nurse, "I am here to save the world. I threw away the pills because they make God go away." The nurse identifies the patient's reason for medication nonadherence as: a. poor alliance with clinicians. b. inadequate discharge planning. c. dislike of medication side effects. d. lack of insight associated with the illness.

ANS: D The patient's nonadherence is most closely related to lack of insight into his illness. The patient believes he is an exalted personage who hears God's voice rather than an individual with a serious mental disorder who needs medication to control his symptoms. While the distracters may play a part in the patient's nonadherence, the correct response is most likely.

A college student said, "Most of the time I'm happy and feel good about myself. I have learned that what I get out of something is proportional to the effort I put into it." Which number on this mental health continuum should the nurse select? Mental Illness Mental Health 1 2 3 4 5 a. 1 b. 2 c. 3 d. 4 e. 5

ANS: E The student is happy and has an adequate self-concept. The student is reality-oriented, works effectively, and has control over own behavior. Mental health does not mean that a person is always happy.

The nurse records this entry in a patient's progress notes: Patient escorted to unit by ER nurse at 2130. Patient's clothing was dirty. In interview room, patient sat with hands over face, sobbing softly. Did not acknowledge nurse or reply to questions. After several minutes, abruptly arose, ran to window, and pounded. Shouted repeatedly, "Let me out of here." Verbal intervention unsuccessful. Order for stat dose 2 mg haloperidol PO obtained; medication administered at 2150. By 2215, patient stopped shouting and returned to sit wordlessly in chair. Patient placed on one-to-one observation. How should this documentation be evaluated? a. Uses unapproved abbreviations b. Contains subjective material c. Too brief to be of value d. Excessively wordy e. Meets standards

ANS: E This narrative note describes patient appearance, behavior, and conversation. It mentions that less-restrictive measures were attempted before administering medication and documents patient response to medication. This note would probably meet standards. A complete nursing assessment would be in order as soon as the patient is able to participate. Subjective material is absent from the note. Abbreviations are acceptable.

An adolescent hospitalized after a violent physical outburst tells the nurse, "I'm going to kill my father, but you can't tell anyone." Select the nurse's best response. A. "You are right. Federal law requires me to keep clinical information private." B. "I am obligated to share that information with the treatment team." C. "Those kinds of thoughts will make your hospitalization longer." D. "You should share this thought with your psychiatrist."

B Breach of nurse-patient confidentiality does not pose a legal dilemma for nurses in these circumstances because a team approach to delivery of psychiatric care presumes communication of patient information to other staff members to develop treatment plans and outcome criteria. The patient should also know that the team has a duty to warn the father of the risk for harm.

Which instruction has priority when teaching a patient about clozapine (Clozaril)? a. "Avoid unprotected sex." b. "Report sore throat and fever immediately." c. "Reduce foods high in polyunsaturated fats." d. "Use over-the-counter preparations for rashes."

B Clozapine therapy may produce agranulocytosis; therefore, signs of infection should be immediately reported to the health care provider. In addition, the patient should have white blood cell levels measured weekly. The other options are not relevant to clozapine.

By which mechanism do selective serotonin reuptake inhibitors (SSRI) improve depression? a. Destroying increased amounts of serotonin b. Making more serotonin available at the synaptic gap c. Increasing production of acetylcholine and dopamine d. Blocking muscarinic and 1 norepinephrine receptors

B Depression is thought to be related to lowered availability of the neurotransmitter serotonin. SSRIs act by blocking reuptake of serotonin, leaving a higher concentration available at the synaptic cleft. SSRIs prevent destruction of serotonin. SSRIs have little or no effect on acetylcholine and dopamine production. SSRIs do not produce muscarinic or 1 norepinephrine blockade.

Which nursing intervention demonstrates false imprisonment? A. A confused and combative patient says, "I'm getting out of here, and no one can stop me." The nurse restrains this patient without a health care provider's order and then promptly obtains an order. B. A patient has been irritating and attention-seeking much of the day. A nurse escorts the patient down the hall saying, "Stay in your room, or you'll be put in seclusion." C. An involuntarily hospitalized patient with suicidal ideation runs out of the psychiatric unit. The nurse rushes after the patient and convinces the patient to return to the unit. D. An involuntarily hospitalized patient with homicidal ideation attempts to leave the facility. A nurse calls the security team and uses established protocols to prevent the patient from leaving.

B False imprisonment involves holding a competent person against his or her will. Actual force is not a requirement of false imprisonment. The individual needs only to be placed in fear of imprisonment by someone who has the ability to carry out the threat. If a patient is not competent (confused), then the nurse should act with beneficence. Patients admitted involuntarily should not be allowed to leave without permission of the treatment team.

The therapeutic action of neurotransmitter inhibitors that block reuptake cause: a. decreased concentration of the blocked neurotransmitter in the central nervous system. b. increased concentration of the blocked neurotransmitter in the synaptic gap. c. destruction of receptor sites specific to the blocked neurotransmitter. d. limbic system stimulation.

B If the reuptake of a substance is inhibited, it accumulates in the synaptic gap, and its concentration increases, permitting ease of transmission of impulses across the synaptic gap. Normal transmission of impulses across synaptic gaps is consistent with normal rather than depressed mood. The other options are not associated with blocking neurotransmitter reuptake.

A nurse would anticipate that treatment for a patient with memory difficulties might include medications designed to: a. inhibit gamma-aminobutyric acid (GABA) b. prevent destruction of acetylcholine c. reduce serotonin metabolism d. increase dopamine activity

B Increased acetylcholine plays a role in learning and memory. Preventing destruction of acetylcholine by acetylcholinesterase would result in higher levels of acetylcholine, with the potential for improved memory. GABA affects anxiety rather than memory. Increased dopamine would cause symptoms associated with schizophrenia or mania rather than improve memory. Decreasing dopamine at receptor sites is associated with Parkinson's disease rather than improving memory.

A patient diagnosed with schizophrenia believes a local minister stirred evil spirits. The patient threatens to bomb a local church. The psychiatrist notifies the minister. Select the answer with the correct rationale. The psychiatrist: A. released information without proper authorization. B. demonstrated the duty to warn and protect. C. violated the patient's confidentiality. D. avoided charges of malpractice.

B It is the health care professional's duty to warn or notify an intended victim after a threat of harm has been made. Informing a potential victim of a threat is a legal responsibility of the health care professional. It is not a violation of confidentiality.

A patient taking medication for mental illness develops restlessness and an uncontrollable need to be in motion. Which drug action causes these symptoms to develop? a. Anticholinergic effects b. Dopamine-blocking effects c. Endocrine-stimulating effects d. Ability to stimulate spinal nerves

B Medication that blocks dopamine often produces disturbances of movement, such as akathisia, because dopamine affects neurons involved in both thought processes and movement regulation. Anticholinergic effects include dry mouth, blurred vision, urinary retention, and constipation. Akathisia is not caused by endocrine stimulation or spinal nerve stimulation.

The nurse prepares to assess a patient diagnosed with major depression for disturbances in circadian rhythms. Which question should the nurse ask this patient? a. "Have you ever seen or heard things that others do not?" b. "What are your worst and best times of the day?" c. "How would you describe your thinking?" d. "Do you think your memory is failing?"

B Mood changes throughout the day may be related to circadian rhythm disturbances. Questions about sleep pattern are also relevant to circadian rhythms. The distracters apply to assessment for illusions and hallucinations, thought processes, and memory.

A patient has fear as well as increased heart rate and blood pressure. The nurse suspects increased activity of which neurotransmitter? a. Gamma-aminobutyric acid (GABA) b. Norepinephrine c. Acetylcholine d. Histamine

B Norepinephrine is the neurotransmitter associated with sympathetic nervous system stimulation, preparing the individual for "fight or flight." GABA is a mediator of anxiety level. A high concentration of histamine is associated with an inflammatory response. A high concentration of acetylcholine is associated with parasympathetic nervous system stimulation.

Two hospitalized patients fight whenever they are together. During a team meeting, a nurse asserts that safety is of paramount importance, so treatment plans should call for both patients to be secluded to keep them from injuring each other. This assertion: A.reinforces the autonomy of the two patients. B.violates the civil rights of both patients. C.represents the intentional tort of battery. D.correctly places emphasis on safety.

B Patients have a right to treatment in the least restrictive setting. Safety is important, but less restrictive measures should be tried first. Unnecessary seclusion may result in a charge of false imprisonment. Seclusion violates the patient's autonomy. The principle by which the nurse is motivated is beneficence, not justice. The tort represented is false imprisonment.

A patient experiencing psychosis asks a psychiatric technician, "What's the matter with me?" The technician replies, "Nothing is wrong with you. You just need to use some self-control." The nurse who overheard the exchange should take action based on: A. the technician's unauthorized disclosure of confidential clinical information. B. violation of the patient's right to be treated with dignity and respect. C. the nurse's obligation to report caregiver negligence. D. the patient's right to social interaction.

B Patients have the right to be treated with dignity and respect. The technician's comment disregards the seriousness of the patient's illness. The Code of Ethics for Nurses requires intervention. Patient emotional abuse has been demonstrated, not negligence. An interaction with the technician is not an aspect of social interaction. The technician did not disclose clinical information.

A nurse prepares to administer a scheduled injection of haloperidol decanoate (Haldol depot) to an outpatient with schizophrenia. As the nurse swabs the site, the patient shouts, "Stop! I don't want to take that medicine anymore. I hate the side effects." Select the nurse's best action. A. Assemble other staff for a show of force and proceed with the injection, using restraint if necessary. B. Stop the medication administration procedure and say to the patient, "Tell me more about the side effects you've been having." C. Proceed with the injection but explain to the patient that there are medications that will help reduce the unpleasant side effects. D. Say to the patient, "Since I've already drawn the medication in the syringe, I'm required to give it, but let's talk to the doctor about delaying next month's dose."

B Patients with mental illness retain their civil rights unless there is clear, cogent, and convincing evidence of dangerousness. The patient in this situation presents no evidence of dangerousness. The nurse, as an advocate and educator, should seek more information about the patient's decision and not force the medication.

Questions the nurse could ask that would be nonjudgmental when obtaining information about patient use of herbal remedies include: (select all that apply) a. "You don't regularly take herbal remedies, do you?" b. "What herbal medicines have you used to relieve your symptoms?" c. "What over-the-counter medicines and nutritional supplements do you use?" d. "What differences in your symptoms do you notice when you take herbal supplements?" e. "Have you experienced problems from using herbal and prescription drugs at the same time?"

B, C, D, E The correct responses are neutral in tone and do not express bias for or against the use of herbal medicines. The distracter, worded in a negative way, makes the nurse's bias evident.

Which action by a nurse constitutes a breach of a patient's right to privacy? A. Documenting the patient's daily behavior during hospitalization B. Releasing information to the patient's employer without consent C. Discussing the patient's history with other staff during care planning D. Asking family to share information about a patient's pre-hospitalization behavior

B Release of information without patient authorization violates the patient's right to privacy. The other options are acceptable nursing practices. See relationship to audience response question.

A nurse caring for a patient taking a selective serotonin reuptake inhibitor (SSRI) will develop outcome criteria related to: a. coherent thought processes b. improvement in depression c. reduced levels of motor activity d. decreased extrapyramidal symptoms

B SSRIs affect mood, relieving depression in many cases. SSRIs do not act to reduce thought disorders. SSRIs reduce depression but have little effect on motor hyperactivity. SSRIs do not produce extrapyramidal symptoms.

A drug blocks the attachment of norepinephrine to 1 receptors. The patient may experience: a. hypertensive crisis b. orthostatic hypotension c. severe appetite disturbance. d. an increase in psychotic symptoms

B Sympathetic-mediated vasoconstriction is essential for maintaining normal blood pressure in the upright position. Blockage of 1 receptors leads to vasodilation and orthostatic hypotension. Orthostatic hypotension may cause fainting and falls. Teach patients ways of minimizing this phenomenon.

A patient has disorganized thinking associated with schizophrenia. Neuroimaging would likely show dysfunction in which part of the brain? a. Hippocampus b. Frontal lobe c. Cerebellum d. Brainstem

B The frontal lobe is responsible for intellectual functioning. The hippocampus is involved in emotions and learning. The cerebellum regulates skeletal muscle coordination and equilibrium. The brainstem regulates internal organs.

Consider these medications: carbamazepine (Tegretol), lamotrigine (Lamictal), gabapentin (Neurontin). Which medication below also belongs with this group? a. Galantamine (Reminyl) b. Valproate (Depakote) c. Buspirone (BuSpar) d. Tacrine (Cognex)

B The medications listed in the stem are mood stabilizers, anticonvulsant types. Valproate (Depakote) is also a member of this group. The distracters are drugs for treatment of Alzheimer's disease and anxiety.

A nurse finds a psychiatric advance directive in the medical record of a patient experiencing psychosis. The directive was executed during a period when the patient was stable and competent. The nurse should: A.review the directive with the patient to ensure it is current. B.ensure that the directive is respected in treatment planning. C.consider the directive only if there is a cardiac or respiratory arrest. D.encourage the patient to revise the directive in light of the current health problem.

B The nurse has an obligation to honor the right to self-determination. An advanced psychiatric directive supports that goal. Since the patient is currently psychotic, the terms of the directive now apply.

An aide in a psychiatric hospital says to the nurse, "We don't have time every day to help each patient complete a menu selection. Let's tell dietary to prepare popular choices and send them to our unit." Select the nurse's best response. A. "Thanks for the suggestion, but that idea may not work because so many patients take MAOI (monoamine oxidase inhibitor) antidepressants." B. "Thanks for the idea, but it's important to treat patients as individuals. Giving choices is one way we can respect patients' individuality." C. "Thank you for the suggestion, but the patients' bill of rights requires us to allow patients to select their own diet." D. "Thank you. That is a very good idea. It will make meal preparation easier for the dietary department."

B The nurse's response to the aide should recognize patients' rights to be treated with dignity and respect as well as promote autonomy. This response also shows respect for the aide and fulfills the nurse's obligation to provide supervision of unlicensed personnel. The incorrect responses have flawed rationale or do not respect patients as individuals.

The parent of an adolescent diagnosed with schizophrenia asks the nurse, "My child's doctor ordered a PET. What kind of test is that?" Select the nurse's best reply. a. "This test uses a magnetic field and gamma waves to identify problem areas in the brain. Does your teenager have any metal implants?" b. "PET means positron-emission tomography. It is a special type of scan that shows blood flow and activity in the brain." c. "A PET scan passes an electrical current through the brain and shows brain-wave activity. It can help diagnose seizures." d."It's a special x-ray that shows structures of the brain and whether there has ever been a brain injury."

B The parent is seeking information about PET scans. It is important to use terms the parent can understand, so the nurse should identify what the initials mean. The correct response is the only option that provides information relevant to PET scans. The distracters describe MRI, CT scans, and EEG. See relationship to audience response question.

A professional football player is seen in the emergency department after losing consciousness from an illegal block. Prior to discharge, the nurse assists the patient to schedule an outpatient computed tomography (CT) scan for the next day. Which strategy should the nurse use to ensure the patient remembers the appointment? a. Write the appointment day, time, and location on a piece of paper and give it to the player. b. Log the appointment day, time, and location into the player's cell phone calendar feature. c. Ask the health care provider to admit the patient to the hospital overnight. d. Verbally inform the patient of the appointment day, time, and location.

B This player may have suffered repeated head injuries with damage to the hippocampus. The hippocampus has significant role in maintaining memory. Logging the appointment into the player's cell phone calendar will remind him of the appointment the next day. Paper will be lost, and the patient is unlikely to remember verbal instruction. Hospitalization is unnecessary. See relationship to audience response question. Caution: This question requires students to apply previous learning regarding central nervous system anatomy and physiology.

A patient diagnosed with bipolar disorder has an unstable mood, aggressiveness, agitation, talkativeness, and irritability. The nurse expects the health care provider to prescribe a medication from which group? a. Psychostimulants b. Mood stabilizers c. Anticholinergics d. Antidepressants

B The symptoms describe mania, which is effectively treated by mood stabilizers, such as lithium, and selected anticonvulsants (carbamazepine, valproic acid, and lamotrigine). Drugs from the other classifications listed are not effective in the treatment of mania.

Which assessment question would produce data that would help a nurse understand healing options acceptable to a client of a different culture? A. "Is there someone in your community who usually cures your illness?" B. "What usually helps people who have the same type of illness you have?" C. "What questions would you like to ask about your condition?" D. "What sorts of stress are you presently experiencing?"

B. Asking about typical treatment seeks information about the "usual" cultural treatment of the disorder experienced by the client.

Ms. Wong, aged 52 years, comes to the emergency room with severe anxiety. She was raised in China but immigrated to the United States at age 40 years. She was recently fired from her job because of a major error in the accounting department that she managed. Ms. Wong's aged parents live with her. Ms. Wong states, "I am a failure." Which of the following statements may accurately assess the basis for Ms. Wong's anxiety and feelings of failure? A. Ms. Wong may feel that she has let herself down since she did not achieve her personal goals in the workplace. B. Ms. Wong may feel that she has shamed the family by being fired and may no longer be able to provide for them. C. Ms. Wong may feel personally inadequate since she failed in her quest for independence and self-reliance. D. Ms. Wong may be feeling anxiety because in her family's traditions her failure may result in a changed fate.

B. Eastern tradition, such as in China, where Ms. Wong is from, sees the family as the basis for one's identity, and family interdependence as the norm. The views expressed in options a and c demonstrate Western tradition where self-reliance, individuality, and autonomy are highly valued. In the Eastern view one is born into an unchangeable fate.

A client reporting gastric pain, tells the nurse, "I think my symptoms started when a neighbor cast a spell on me." The assessment the nurse can make is that the client A. has a major mental illness. B. is expressing a culture-bound illness. C. requires hospitalization to protect the neighbor. D. will probably not respond to Western medical treatment.

B. Many culture-bound illnesses, such as ghost illness, or hwa byung, seem exotic or irrational to American nurses. Many of these illnesses cannot be understood within a Western medicine framework. Their causes, manifestations, and treatments do not make sense to nurses whose understanding is limited to a Western perspective on disease and illness.

Data concerning client age, sex, education, and income should be the focus of an assessment in order to best understand cultural issues related to A. health practices. B. power and control. C. psychological stability. D. assimilation and conformity.

B. Power and control are often products of culturally determined beliefs about who should hold power. In many cultures the elderly are venerated. In other cultures women are virtually powerless. For some cultures, higher education equates with power.

The Eastern world view can be identified by the belief that A. one's identity is found in individuality. B. holds responsibility to family as central. C. time waits for no one. D. disease is a lack of harmony with the environment.

B. The Eastern traditional world view is sociocentric. Individuals experience their selfhood and their lives as part of an interdependent web of relationships and expectations.

A voluntarily hospitalized patient tells the nurse, "Get me the forms for discharge. I want to leave now." Select the nurse's best response. A. "I will get the forms for you right now and bring them to your room." B. "Since you signed your consent for treatment, you may leave if you desire." C. "I will get them for you, but let's talk about your decision to leave treatment." D. "I cannot give you those forms without your health care provider's permission."

C A voluntarily admitted patient has the right to demand and obtain release in most states. However, as a patient advocate, the nurse is responsible for weighing factors related to the patient's wishes and best interests. By asking for information, the nurse may be able to help the patient reconsider the decision. Facilitating discharge without consent is not in the patient's best interests before exploring the reason for the request.

After leaving work, a nurse realizes documentation of administration of a PRN medication was omitted. This off-duty nurse phones the nurse on duty and says, "Please document administration of the medication for me. My password is alpha1." The nurse receiving the call should: A. fulfill the request promptly. B. document the caller's password. C. refer the matter to the charge nurse to resolve. D. report the request to the patient's health care provider.

C Fraudulent documentation may be grounds for discipline by the state board of nursing. Referring the matter to the charge nurse will allow observance of hospital policy while ensuring that documentation occurs. Notifying the health care provider would be unnecessary when the charge nurse can resolve the problem. Nurses should not provide passwords to others.

A nurse can anticipate anticholinergic side effects are likely when a patient takes: a. lithium (Lithobid) b. buspirone (BuSpar) c. imipramine (Tofranil) d. risperidone (Risperdal)

C Imipramine (Tofranil) is a tricyclic antidepressant with strong anticholinergic properties, resulting in dry mouth, blurred vision, constipation, and urinary retention. Lithium therapy is more often associated with fluid-balance problems, including polydipsia, polyuria, and edema. Risperidone therapy is more often associated with movement disorders, orthostatic hypotension, and sedation. Buspirone is associated with anxiety reduction without major side effects.

Which individual diagnosed with a mental illness may need involuntary hospitalization? An individual: A. who has a panic attack after her child gets lost in a shopping mall B. with visions of demons emerging from cemetery plots throughout the community C. who takes 38 acetaminophen tablets after the person's stock portfolio becomes worthless D. diagnosed with major depression who stops taking prescribed antidepressant medication

C Involuntary hospitalization protects patients who are dangerous to themselves or others and cannot care for their own basic needs. Involuntary hospitalization also protects other individuals in society. An overdose of acetaminophen indicates dangerousness to self. The behaviors described in the other options are not sufficient to require involuntary hospitalization.

A nurse instructs a patient taking a drug that inhibits monoamine oxidase (MAO) to avoid certain foods and drugs because of the risk of: a. cardiac dysrhythmia b. hypotensive shock c. hypertensive crisis d. hypoglycemia

C Patients taking MAO-inhibiting drugs must be on a low tyramine diet to prevent hypertensive crisis. In the presence of MAO inhibitors, tyramine is not destroyed by the liver and in high levels produces intense vasoconstriction, resulting in elevated blood pressure.

A nurse cares for a group of patients receiving various medications, including haloperidol (Haldol), carbamazepine (Tegretol), trazodone (Desyrel), and phenelzine (Nardil). The nurse will order a special diet for the patient who takes: a. carbamazepine b. haloperidol c. phenelzine d. trazodone

C Patients taking phenelzine, an MAO inhibitor, must be on a low tyramine diet to prevent hypertensive crisis. There are no specific dietary precautions associated with the distracters.

A patient is hospitalized for severe depression. Of the medications listed below, the nurse can expect to provide the patient with teaching about: a. chlordiazepoxide (Librium) b. clozapine (Clozaril) c. sertraline (Zoloft) d. tacrine (Cognex)

C Sertraline (Zoloft) is an SSRI. This antidepressant blocks the reuptake of serotonin, with few anticholinergic and sedating side effects. Clozapine is an antipsychotic. Chlordiazepoxide is an anxiolytic. Tacrine treats Alzheimer's disease.

A new antidepressant is prescribed for an elderly patient with major depression, but the dose is more than the usual geriatric dose. The nurse should: A. consult a reliable drug reference. B. teach the patient about possible side effects and adverse effects. C. withhold the medication and confer with the health care provider. D. encourage the patient to increase oral fluids to reduce drug concentration.

C The dose of antidepressants for elderly patients is often less than the usual adult dose. The nurse should withhold the medication and consult the health care provider who wrote the order. The nurse's duty is to practice according to professional standards as well as intervene and protect the patient.

In which situations would a nurse have the duty to intervene and report? Select all that apply. A. A peer has difficulty writing measurable outcomes. B. A health care provider gives a telephone order for medication. C. A peer tries to provide patient care in an alcohol-impaired state. D. A team member violates relationship boundaries with a patient. E. A patient refuses medication prescribed by a licensed health care provider.

C, D Both keyed answers are events that jeopardize patient safety. The distracters describe situations that may be resolved with education or that are acceptable practices.

Which actions violate the civil rights of a psychiatric patient? The nurse: (select all that apply) A. performs mouth checks after overhearing a patient say, "I've been spitting out my medication." B. begins suicide precautions before a patient is assessed by the health care provider. C. opens and reads a letter a patient left at the nurse's station to be mailed. D. places a patient's expensive watch in the hospital business office safe. E. restrains a patient who uses profanity when speaking to the nurse.

C, E The patient has the right to send and receive mail without interference. Restraint is not indicated because a patient uses profanity; there are other less restrictive ways to deal with this behavior. The other options are examples of good nursing judgment and do not violate the patient's civil rights.

The psychiatric mental health nurse working with depressed clients of the Eastern culture must realize that a useful outcome criterion might be if client reports A. increased somatic expressions of distress. B. disruption of energy balance. C. appeasement of the spirits. D. increased anxiety.

C. Appeasement of spirits might be a viable outcome criterion if the client believes the illness was caused by angry spirits. In each of the other options useful outcomes would be decreased somatic symptoms, reinstatement of energy balance, and decreased anxiety.

Which of the following best explains the concept of cultural competence? A. Nurses have enough knowledge about different cultures to be assured they are delivering culturally sensitive care. B. Nurses are able to educate their patients from other cultures appropriately about the cultural norms of the United States. C. Nurses adjust their own practices to meet their patients' cultural preferences, beliefs, and practices. D. Nurses must take continuing education classes on culture in the process of becoming culturally competent.

C. Cultural competence means that nurses adjust and conform to their patients' cultural needs, beliefs, practices, and preferences rather than their own. This option does not describe cultural competence. Although nurses are continually learning regarding culture, it is a career-long process. The goal is not to educate patients about our own culture but rather to adjust to their cultural preferences. Although nurses may take continuing education regarding culture, this does not describe the term cultural competence. The other options do not describe cultural competence.

A peer asks you to help him differentiate between culture and ethnicity for clarification. Which statement by the peer would acknowledge that you had appropriately helped him clarify the difference between the two terms? A. "So, ethnicity refers to having the same life goals whereas culture refers to race." B. "So, ethnicity refers to norms within a culture, and culture refers to shared likes and dislikes." C. "So, ethnicity refers to shared history and heritage, whereas culture refers to sharing the same beliefs and values." D. "So, ethnicity refers to race, and culture refers to having the same worldview."

C. Ethnicity is sharing a common history and heritage. Culture comprises the shared beliefs, values, and practices that guide a group's members in patterned ways of thinking and acting. The other options are all incorrect definitions of ethnicity and culture.

Clients of another culture are at greatest risk for misdiagnosis of a psychiatric problem because of A. biased assessment tools. B. insensitive practitioners. C. insensitive interviewing techniques. D. lack of the availability of cultural translators.

C. Inaccurate information or insufficient information may be obtained if the interviewer is not culturally sensitive. Only when assessment data are accurate can effective treatment be planned.

When members of a group are introduced to the culture's worldview, beliefs, values, and practices, it is called A. acculturation. B. ethnocentrism. C. enculturation. D. cultural encounters.

C. Members of a group are introduced to the culture's worldview, beliefs, values, and practices in a process called enculturation. Ethnocentrism is the universal tendency of humans to think that their way of thinking and behaving is the only correct and natural way. Acculturation is learning the beliefs, values, and practices of a new cultural setting, which sometimes takes several generations. Cultural encounters occur when members of varying cultures meet and interact.

Exclusive use of Western psychological theories by nurses making client assessments will result in A. a high level of care for all clients. B. standardization of nomenclature for psychiatric disorders. C. inadequate assessment of clients of diverse cultures. D. greater ease in selecting appropriate treatment interventions.

C. Unless clients have faith in a particular healing modality, the treatment may not be effective. When nurses make assessments on the basis of Western theories, treatments consistent with those assessments follow. Clients of other cultures may find the treatment modalities unacceptable or not useful. Treatments consistent with the client's cultural beliefs as to what will provide a cure are better.

An individual hiking in the forest encounters a large poisonous snake on the path. Which change in this individual's vital signs is most likely? a. Pulse rate changes from 90 to 72. b. Respiratory rate changes from 22 to 18. c. Complaints of intestinal cramping begin. d. Blood pressure changes from 114/62 to 136/78.

D This frightening experience would stimulate the sympathetic nervous system, causing a release of norepinephrine, an excitatory neurotransmitter. It prepares the body for fight or flight. Increased blood pressure, pupil size, respiratory rate, and pulse rate signify release of norepinephrine. Intestinal cramping would be associated with stimulation of the parasympathetic nervous system.

A newly admitted acutely psychotic patient is a private patient of the medical director and a private-pay patient. To whom does the psychiatric nurse assigned to the patient owe the duty of care? A. Medical director B. Hospital C. Profession D. Patient

D Although the nurse is accountable to the health care provider, the agency, the patient, and the profession, the duty of care is owed to the patient.

What is the legal significance of a nurse's action when a patient verbally refuses medication and the nurse gives the medication over the patient's objection? The nurse: A. has been negligent. B. committed malpractice. C. fulfilled the standard of care. D. can be charged with battery.

D Battery is an intentional tort in which one individual violates the rights of another through touching without consent. Forcing a patient to take medication after the medication was refused constitutes battery. The charge of battery can be brought against the nurse. The medication may not necessarily harm the patient; harm is a component of malpractice.

A patient has acute anxiety related to an automobile accident 2 hours ago. The nurse should teach the patient about medication from which group? a. tricyclic antidepressants b. antipsychotic drugs c. antimanic drugs d. benzodiazepines

D Benzodiazepines provide anxiety relief. Tricyclic antidepressants are used to treat symptoms of depression. Antimanic drugs are used to treat bipolar disorder. Antipsychotic drugs are used to treat psychosis.

A patient with psychosis became aggressive, struck another patient, and required seclusion. Select the best documentation. A. Patient struck another patient who attempted to leave day room to go to bathroom. Seclusion necessary at 1415. Plan: Maintain seclusion for 8 hours and keep these two patients away from each other for 24 hours. B. Seclusion ordered by physician at 1415 after command hallucinations told the patient to hit another patient. Careful monitoring of patient maintained during period of seclusion. C. Seclusion ordered by MD for aggressive behavior. Begun at 1415. Maintained for 2 hours without incident. Outcome: Patient calmer and apologized for outburst. D. Patient pacing, shouting. Haloperidol 5 mg given PO at 1300. No effect by 1315. At 1415 patient yelled, "I'll punch anyone who gets near me," and struck another patient with fist. Physically placed in seclusion at 1420. Seclusion order obtained from MD at 1430.

D Documentation must be specific and detail the key aspects of care. It should demonstrate implementation of the least restrictive alternative. Justification for why a patient was secluded should be recorded, along with interventions attempted in an effort to avoid seclusion. Documentation should include a description of behavior and verbalizations, interventions tried and their outcomes, and the name of the health care provider ordering the use of seclusion.

Which individual with mental illness may need emergency or involuntary admission? The individual who: A. resumes using heroin while still taking naltrexone (ReVia). B. reports hearing angels playing harps during thunderstorms. C. does not keep an outpatient appointment with the mental health nurse. D. throws a heavy plate at a waiter at the direction of command hallucinations.

D Throwing a heavy plate is likely to harm the waiter and is evidence of dangerousness to others. This behavior meets the criteria for emergency or involuntary hospitalization for mental illness. The behaviors in the other options evidence mental illness but not dangerousness. See related audience response question.

Which patient meets criteria for involuntary hospitalization for psychiatric treatment? The patient who: A. is noncompliant with the treatment regimen. B. fraudulently files for bankruptcy. C. sold and distributed illegal drugs. D. threatens to harm self and others.

D Involuntary hospitalization protects patients who are dangerous to themselves or others and cannot care for their own basic needs. Involuntary commitment also protects other individuals in society. The behaviors described in the other options are not sufficient to require involuntary hospitalization.

A patient tells the nurse, "My doctor prescribed Paxil (paroxetine) for my depression. I assume I'll have side effects like I had when I was taking Tofranil (imipramine)." The nurse's reply should be based on the knowledge that paroxetine is a(n): a. selective norepinephrine reuptake inhibitor. b. tricyclic antidepressant. c. MAO inhibitor d. SSRI

D Paroxetine is an SSRI and will not produce the same side effects as imipramine, a tricyclic antidepressant. The patient will probably not experience dry mouth, constipation, or orthostatic hypotension.

A patient begins therapy with a phenothiazine medication. What teaching should the nurse provide related to the drug's strong dopaminergic effect? a. Chew sugarless gum b. Increase daily fiber c. Arise slowly from bed d. Report changes in muscle movement

D Phenothiazines block dopamine receptors in both the limbic system and basal ganglia. Movement disorders and motor abnormalities (extrapyramidal side effects), such as parkinsonism, akinesia, akathisia, dyskinesia, and tardive dyskinesia, are likely to occur early in the course of treatment. They are often heralded by sensations of muscle stiffness. Early intervention with antiparkinsonism medication can increase the patient's comfort and prevent dystonic reactions. The distracters are related to anticholinergic effects.

Which documentation of a patient's behavior best demonstrates a nurse's observations? A. Isolates self from others. Frequently fell asleep during group. Vital signs stable. B. Calmer; more cooperative. Participated actively in group. No evidence of psychotic thinking. C. Appeared to hallucinate. Frequently increased volume on television, causing conflict with others. D. Wore four layers of clothing. States, "I need protection from evil bacteria trying to pierce my skin."

D The documentation states specific observations of the patient's appearance and the exact statements made. The other options are vague or subjective statements and can be interpreted in different ways.

A family member of a patient with delusions of persecution asks the nurse, "Are there any circumstances under which the treatment team is justified in violating a patient's right to confidentiality?" The nurse should reply that confidentiality may be breached: A. under no circumstances. B. at the discretion of the psychiatrist. C. when questions are asked by law enforcement. D. if the patient threatens the life of another person.

D The duty to warn a person whose life has been threatened by a psychiatric patient overrides the patient's right to confidentiality. The right to confidentiality is not suspended at the discretion of the therapist or for legal investigations.

A patient's history shows drinking 4 to 6 liters of fluid and eating more than 6,000 calories per day. Which part of the central nervous system is most likely dysfunctional for this patient? a. Amydala b. Parietal lobe c. Hippocampus d. Hypothalamus

D The hypothalamus, a small area in the ventral superior portion of the brainstem, plays a vital role in such basic drives as hunger, thirst, and sex.

In a team meeting a nurse says, "I'm concerned about whether we are behaving ethically by using restraint to prevent one patient from self-mutilation, while the care plan for another self-mutilating patient requires one-on-one supervision." Which ethical principle most clearly applies to this situation? A.Beneficence B.Autonomy C.Fidelity D.Justice

D The nurse is concerned about justice, that is, fair distribution of care, which includes treatment with the least restrictive methods for both patients. Beneficence means promoting the good of others. Autonomy is the right to make one's own decisions. Fidelity is the observance of loyalty and commitment to the patient.

1. A patient asks, "What are neurotransmitters? The doctor said mine are imbalanced." Select the nurse's best response. a. "How do you feel about having imbalanced neurotransmitters?" b. "Neurotransmitters protect us from harmful effects of free radicals." c. "Neurotransmitters are substances we consume that influence memory and mood." d. "Neurotransmitters are natural chemicals that pass messages between brain cells."

D The patient asked for information, and the correct response is most accurate. Neurotransmitters are chemical substances that function as messengers in the central nervous system. They are released from the axon terminal, diffuse across the synapse, and attach to specialized receptors on the postsynaptic neuron. The distracters either do not answer the patient's question or provide untrue, misleading information.

Deviation from cultural expectations is considered by members of the cultural group as a demonstration of A. hostility. B. lack of self-will. C. variation from tradition. D. illness.

D. Deviation from cultural expectations is considered by others in the culture to be a problem and is frequently defined by the cultural group as "illness."

When assessing and planning treatment for a client who has recently arrived in the United States from China, the nurse should be alert to the possibility that the client's explanatory model for his illness reflects A. supernatural causes. B. negative forces. C. inheritance. D. imbalance.

D. Many Eastern cultures explain illness as a function of imbalance.

Which healing practice is least used in the Western health system of healing practices? A. Antibiotic medication B. Surgery C. Targeted cellular destruction D. Restoring lost balance or harmony

D. The best treatment perspectives of various cultures include regaining lost balance and harmony. This perspective is not used in Western culture.

The question that would give data of least value to the assessment of family dynamics is A. "What changes have occurred recently at work?" B. "Are your wife and children conforming to your expectations?" C. "Are you experiencing stress associated with conforming to family expectations?" D. "Do you expect others to shun or avoid you because you are seeing a therapist?"

D. The question about others' reaction to seeking help from a psychotherapist will not provide data about family dynamics.

You are working on the psychiatric unit and assisting with the care for Mr. Tran, a refugee from Darfur, who came to the United States 1 year ago. Although Mr. Tran understands and speaks some very limited English, he is much more comfortable conversing in his native language. Mike, the nurse working directly with Mr. Tran, says to you, "I am so frustrated trying to communicate with Mr. Tran! He insists on speaking his language instead of English. I think if people want to live here, they ought to have to speak our language and act like we do!" Which of the following responses you could make promotes culturally competent care? (select all that apply): A. "You are right that Mr. Tran needs to speak English, but all patients do have a right to an interpreter, so you need to comply." B. "I agree that it is frustrating trying to communicate with Mr. Tran. Maybe we could see if his family members can help convince him to try speaking English." C. "Mr. Tran will have to learn to speak English eventually to live and work successfully in this country. Just try to be patient and encourage him to try speaking English." D. "What you are saying is actually considered cultural imposition, which is imposing our own culture onto someone from a different culture." E. "Mr. Tran's ability to speak and understand English is very limited. He needs to have an interpreter to make sure he can make his needs and feelings known."

D. E. Cultural imposition is imposing our own cultural norms onto those from another cultural group. By obtaining an interpreter for Mr. Tran, the nurse is promoting culturally competent care, ensuring the patient can communicate his feelings and needs thoroughly to the staff. Patients do have a right to an interpreter, but stating that Mike is right is not promoting culturally competent care and is instead confirming his opinion. Asking family members to convince the patient to speak English is not promoting culturally competent care and also undermines the trust between nurse and patient. Instead of encouraging the patient to speak English an interpreter should be obtained for the patient.

32. A patient receiving risperidone (Risperdal) reports severe muscle stiffness at 1030. By 1200, the patient has difficulty swallowing and is drooling. By 1600, vital signs are 102.8° F; pulse 110; respirations 26; 150/90. The patient is diaphoretic. Select the nurse's best analysis and action. a. Agranulocytosis; institute reverse isolation. b. Tardive dyskinesia; withhold the next dose of medication. c. Cholestatic jaundice; begin a high-protein, high-cholesterol diet. d. Neuroleptic malignant syndrome; notify health care provider stat.

NS: D Taking an antipsychotic medication coupled with the presence of extrapyramidal symptoms, such as severe muscle stiffness and difficulty swallowing, hyperpyrexia, and autonomic symptoms (pulse elevation), suggest neuroleptic malignant syndrome, a medical emergency. The symptoms given in the scenario are not consistent with the medical problems listed in the incorrect options.


Conjuntos de estudio relacionados

HRIM 442 Ch 17 Exam 3, Marketing Ch 17, Marketing Ch 17-19, Marketing Chapter 17 & 18, Marketing Chapter 17, mkt ch 16, Marketing 4, MKT 301 - Ch. 16, Marketing Chapter 17, Marketing Chapters 16-18, mktg ch 10, MKTG Ch.10, CH.10-, Marketing Exam 3 Qu...

View Set

AP Biology - Chemistry of Life - Unit I

View Set

LEGAL ENVIRONMENT - CHAPTERS 8 & 9

View Set

[FINAL] Bacterial Genetics - Mastering Microbiology

View Set

What IS or is NOT Differentiate Instruction

View Set

pituitary and adrenal glands - chapter 62

View Set